Ch 12: Pregnancy at Risk: Gestational Conditions, Chapter 11: High Risk Perinatal Care: Preexisting Conditions, Chapter 10: Assessment of High Risk Pregnancy, Maternal Child Nursing Care - Chapter 9: Maternal and Fetal Nutrition, Chapter 8: Nursing C...

अब Quizwiz के साथ अपने होमवर्क और परीक्षाओं को एस करें!

A man's wife is pregnant for the third time. One child was born with cystic fibrosis, and the other child is healthy. The man wonders what the chance is that this child will have cystic fibrosis. This type of testing is known as: a. Occurrence risk. c. Predictive testing. b. Recurrence risk. d. Predisposition testing.

B

9. A woman is undergoing a nipple-stimulated contraction stress test (CST). She is having contractions that occur every 3 minutes. The fetal heart rate (FHR) has a baseline of approximately 120 beats/min without any decelerations. The interpretation of this test is said to be: a. Negative. b. Positive. c. Satisfactory. d. Unsatisfactory.

a. Negative.

The nurse must be cognizant that an individual's genetic makeup is known as his or her: a. Genotype. b. Karyotype. c. Phenotype. d. Chromotype.

A

The placenta allows exchange of oxygen, nutrients, and waste products between the mother and fetus by: a. Contact between maternal blood and fetal capillaries within the chorionic villi. b. Interaction of maternal and fetal pH levels within the endometrial vessels. c. A mixture of maternal and fetal blood within the intervillous spaces. d. Passive diffusion of maternal carbon dioxide and oxygen into the fetal capillaries.

A

A key finding from the Human Genome Project is: a. Approximately 20,000 genes make up the genome. b. All human beings are 80.99% identical at the DNA level. c. Human genes produce only one protein per gene; other mammals produce three proteins per gene. d. Single gene testing will become a standardized test for all pregnant clients in the future.

A

A maternity nurse should be aware of which fact about the amniotic fluid? a. It serves as a source of oral fluid and a repository for waste from the fetus. b. The volume remains about the same throughout the term of a healthy pregnancy. c. A volume of less than 300 mL is associated with gastrointestinal malformations. d. A volume of more than 2 L is associated with fetal renal abnormalities.

A

A new mother asks the nurse about the "white substance" covering her infant. The nurse explains that the purpose of vernix caseosa is to: a. Protect the fetal skin from amniotic fluid. b. Promote normal peripheral nervous system development. c. Allow transport of oxygen and nutrients across the amnion. d. Regulate fetal temperature.

A

A woman asks the nurse, "What protects my baby's umbilical cord from being squashed while the baby's inside of me?" The nurse's best response is: a. "Your baby's umbilical cord is surrounded by connective tissue called Wharton jelly, which prevents compression of the blood vessels and ensures continued nourishment of your baby." b. "Your baby's umbilical floats around in blood anyway." c. "You don't need to worry about things like that." d. "The umbilical cord is a group of blood vessels that are very well protected by the placenta."

A

Many parents-to-be have questions about multiple births. Maternity nurses should be able to tell them that: a. Twinning and other multiple births are increasing because of the use of fertility drugs and delayed childbearing. b. Dizygotic twins (two fertilized ova) have the potential to be conjoined twins. c. Identical twins are more common in white families. d. Fraternal twins are same gender, usually male.

A

The nurse caring for a pregnant client knows that her health teaching regarding fetal circulation has been effective when the client reports that she has been sleeping: a. In a side-lying position. b. On her back with a pillow under her knees. c. With the head of the bed elevated. d. On her abdomen.

A

The nurse caring for the laboring woman should know that meconium is produced by: a. Fetal intestines. b. Amniotic fluid. c. Fetal kidneys. d. The placenta.

A

The nurse is assessing the knowledge of new parents with a child born with maple syrup urine disease (MSUD). This is an autosomal recessive inherited disorder, which means that: a. Both genes of a pair must be abnormal for the disorder to be expressed. b. Only one copy of the abnormal gene is required for the disorder to be expressed. c. The disorder occurs in males and heterozygous females. d. The disorder is carried on the X chromosome.

A

With regard to prenatal genetic testing, nurses should be aware that: a. Maternal serum screening can determine whether a pregnant woman is at risk of carrying a fetus with Down syndrome. b. Carrier screening tests look for gene mutations of people already showing symptoms of a disease. c. Predisposition testing predicts with near certainty that symptoms will appear. d. Presymptomatic testing is used to predict the likelihood of breast cancer.

A

As relates to the father's acceptance of the pregnancy and preparation for childbirth, the maternity nurse should know that: A. The father goes through three phases of acceptance of his own. B. The father's attachment to the fetus cannot be as strong as that of the mother because it does not start until after birth. C. In the last 2 months of pregnancy, most expectant fathers suddenly get very protective of their established lifestyle and resist making changes to the home. D. Typically men remain ambivalent about fatherhood right up to the birth of their child.

A (A father typically goes through three phases of development to reach acceptance of fatherhood: the announcement phase, the moratorium phase, and the focusing phase. The father-child attachment can be as strong as the mother-child relationship and can also begin during pregnancy. In the last 2 months of pregnancy, many expectant fathers work hard to improve the environment of the home for the child. Typically the expectant father's ambivalence ends by the first trimester, and he progresses to adjusting to the reality of the situation and then to focusing on his role.)

Which statement about pregnancy is accurate? A. A normal pregnancy lasts about 10 lunar months. B. A trimester is one third of a year. C. The prenatal period extends from fertilization to conception. D. The estimated date of confinement (EDC) is how long the mother will have to be bedridden after birth.

A (A lunar month lasts 28 days, or 4 weeks. Pregnancy spans 9 calendar months but 10 lunar months. A trimester is one third of a normal pregnancy, or about 13 to 14 weeks. The prenatal period covers the full course of pregnancy (prenatal means before birth). The EDC is now called the EDB, or estimated date of birth. It has nothing to do with the duration of bed rest.)

With regard to medications, herbs, shots, and other substances normally encountered by pregnant women, the maternity nurse should be aware that: A. Both prescription and over-the-counter (OTC) drugs that otherwise are harmless can be made hazardous by metabolic deficiencies of the fetus. B. The greatest danger of drug-caused developmental deficits in the fetus is seen in the final trimester. C. Killed-virus vaccines (e.g., tetanus) should not be given during pregnancy, but live-virus vaccines (e.g., measles) are permissible. D. No convincing evidence exists that secondhand smoke is potentially dangerous to the fetus.

A (Both prescription and OTC drugs that otherwise are harmless can be made hazardous by metabolic deficiencies of the fetus. This is especially true for new medications and combinations of drugs. The greatest danger of drug-caused developmental defects exists in the interval from fertilization through the first trimester, when a woman may not realize that she is pregnant. Live-virus vaccines should be part of postpartum care; killed-virus vaccines may be administered during pregnancy. Secondhand smoke is associated with fetal growth restriction and increases in infant mortality.)

Prenatal testing for human immunodeficiency virus (HIV) is recommended for: A. All women, regardless of risk factors. B. A woman who has had more than one sexual partner. C. A woman who has had a sexually transmitted infection. D. A woman who is monogamous with her partner.

A (Testing for the antibody to HIV is strongly recommended for all pregnant women. A HIV test is recommended for all women, regardless of risk factors. Women who test positive for HIV can be treated, reducing the risk of transmission to the fetus.)

Which behavior indicates that a woman is "seeking safe passage" for herself and her infant? A. She keeps all prenatal appointments. B. She drives her car slowly. C. She "eats for two." D. She wears only low-heeled shoes.

A (The goal of prenatal care is to foster a safe birth for the infant and mother. Although eating properly, driving carefully, and using proper body mechanics all are healthy measures that a mother can take, obtaining prenatal care is the optimal method for providing safety for both herself and her baby)

While teaching the expectant mother about personal hygiene during pregnancy, maternity nurses should be aware that: A. Tub bathing is permitted even in late pregnancy unless membranes have ruptured. B. The perineum should be wiped from back to front. C. Bubble bath and bath oils are permissible because they add an extra soothing and cleansing action to the bath. D. Expectant mothers should use specially treated soap to cleanse the nipples.

A (The main danger from taking baths is falling in the tub. The perineum should be wiped from front to back. Bubble baths and bath oils should be avoided because they may irritate the urethra. Soap, alcohol, ointments, and tinctures should not be used to cleanse the nipples because they remove protective oils. Warm water is sufficient.)

The multiple marker test is used to assess the fetus for which condition? A. Down syndrome B. Congenital cardiac abnormality C. Diaphragmatic hernia D. Anencephaly

A (The maternal serum level of alpha-fetoprotein is used to screen for Down syndrome, neural tube defects, and other chromosome anomalies. The multiple marker test would not detect diaphragmatic hernia, congenital cardiac abnormality, or anencephaly. Additional testing, such as ultrasonography and amniocentesis, would be required to diagnose these conditions.)

With regard to medications, herbs, shots, and other substances normally encountered, the maternity nurse should be aware that: A. Prescription and over-the-counter (OTC) drugs that otherwise are harmless can be made hazardous by metabolic deficiencies of the fetus. B. The greatest danger of drug-caused developmental deficits in the fetus is seen in the final trimester. C. Killed-virus vaccines (e.g., tetanus) should not be given during pregnancy, but live-virus vaccines (e.g., measles) are permissible. D. No convincing evidence exists that secondhand smoke is potentially dangerous to the fetus.

A (This is especially true for new medications and combinations of drugs. The greatest danger of drug-caused developmental defects exists in the interval from fertilization through the first trimester, when a woman may not realize that she is pregnant. Live-virus vaccines should be part of postpartum care; killed-virus vaccines may be administered during pregnancy. Secondhand smoke is associated with fetal growth restriction and increases in infant mortality.)

Which nutritional recommendation about fluids is accurate? A) A woman's daily intake should be eight to ten glasses (2.3 L) of water, milk, or juice. B) Coffee should be limited to no more than two cups, but tea and cocoa can be consumed without worry. C) Of the artificial sweeteners, only aspartame has not been associated with any maternity health concerns. D) Water with fluoride is especially encouraged because it reduces the child's risk of tooth decay.

A) A woman's daily intake should be eight to ten glasses (2.3 L) of water, milk, or juice.

Which statement regarding acronyms in nutrition is accurate? A) Dietary reference intakes (DRIs) consist of recommended dietary allowances (RDAs), adequate intakes (AIs), and upper limits (ULs). B) RDAs are the same as ULs except with better data. C) AIs offer guidelines for avoiding excessive amounts of nutrients. D) They all refer to green leafy vegetables, whole grains, and fruit.

A) Dietary reference intakes (DRIs) consist of recommended dietary allowances (RDAs), adequate intakes (AIs), and upper limits (ULs).

Which nutrients recommended dietary allowance (RDA) is higher during lactation than during pregnancy? A) Energy (kcal) B) Iron C) Vitamin A D) Folic acid

A) Energy (kcal)

Identify a goal of a patient with the following nursing diagnosis: Imbalanced Nutrition: Less Than Body Requirements related to diet choices inadequate to meet nutrient requirements of pregnancy. A) Gain a total of 30 lb. B) Take daily supplements consistently. C) Decrease intake of snack foods. D) Increase intake of complex carbohydrates.

A) Gain a total of 30 lb.

A pregnant patient would like to know a good food source of calcium other than dairy products. Your best answer is: A) Legumes B) Yellow vegetables C) Lean meat D) Whole grains

A) Legumes

With regard to protein in the diet of pregnant women, nurses should be aware that: A) Many protein-rich foods are also good sources of calcium, iron, and B vitamins. B) Many women need to increase their protein intake during pregnancy. C) As with carbohydrates and fat, no specific recommendations exist for the amount of protein in the diet. D) High-protein supplements can be used without risk by women on macrobiotic diets.

A) Many protein-rich foods are also good sources of calcium, iron, and B vitamins.

Most women with uncomplicated pregnancies can use the nurse as their primary source for nutritional information. The nurse or midwife should refer a client to a registered dietitian for in-depth nutritional counseling in the following situations (Select all that apply). A) Preexisting or gestational illness such as diabetes B) Ethnic or cultural food patterns C) Obesity D) Vegetarian diet E) Allergy to tree nuts

A) Preexisting or gestational illness such as diabetes B) Ethnic or cultural food patterns C) Obesity D) Vegetarian diet

After you complete your nutritional counseling for a pregnant woman, you ask her to repeat your instructions so you can assess her understanding of the instructions given. Which statement indicates that she understands the role of protein in her pregnancy? A) Protein will help my baby grow. B) Eating protein will prevent me from becoming anemic. C) Eating protein will make my baby have strong teeth after he is born. D) Eating protein will prevent me from being diabetic.

A) Protein will help my baby grow.

A pregnant woman reports that she is still playing tennis at 32 weeks of gestation. The nurse would be most concerned that during and after tennis matches this woman consumes: A) Several glasses of fluid. B) Extra protein sources such as peanut butter. C) Salty foods to replace lost sodium. D) Easily digested sources of carbohydrate.

A) Several glasses of fluid.

To help a woman reduce the severity of nausea caused by morning sickness, the nurse might suggest that she: A) Try a tart food or drink such as lemonade or salty foods such as potato chips. B) Drink plenty of fluids early in the day. C) Brush her teeth immediately after eating. D) Never snack before bedtime.

A) Try a tart food or drink such as lemonade or salty foods such as potato chips.

Signs and symptoms that a woman should report immediately to her health care provider include (Select all that apply): A. Vaginal bleeding. B. Rupture of membranes. C. Heartburn accompanied by severe headache. D. Decreased libido. E. Urinary frequency.

A, B, C (Vaginal bleeding, rupture of membranes, and severe headaches all are signs of potential complications in pregnancy. Clients should be advised to report these signs to the health care provider. Decreased libido and urinary frequency are common discomforts of pregnancy that do not require immediate health care interventions.)

26. Intrauterine growth restriction (IUGR) is associated with numerous pregnancy-related risk factors (Select all that apply). a. Poor nutrition b. Maternal collagen disease c. Gestational hypertension d. Premature rupture of membranes e. Smoking

A, B, C, E

27. Transvaginal ultrasonography is often performed during the first trimester. While preparing your 6-week gestation patient for this procedure, she expresses concerns over the necessity for this test. The nurse should explain that this diagnostic test may be indicated for a number of situations (Select all that apply). a. Multifetal gestation b. Obesity c. Fetal abnormalities d. Amniotic fluid volume e. Ectopic pregnancy

A, B, C, E

A woman has just moved to the United States from Mexico. She is 3 months pregnant and has arrived for her first prenatal visit. During her assessment interview, you discover that she has not had any immunizations. Which immunizations should she receive at this point in her pregnancy (Select all that apply)? A. Tetanus B. Diphtheria C. Chickenpox D. Rubella E. Hepatitis B

A, B, E (Immunization with live or attenuated live viruses is contraindicated during pregnancy because of potential teratogenicity. Vaccines consisting of killed viruses may be used. Immunizations that may be administered during pregnancy include tetanus, diphtheria, recombinant hepatitis B, and rabies vaccines. Live-virus vaccines include those for measles (rubeola and rubella), chickenpox, and mumps.)

Which statements about multifetal pregnancy are most appropriate? (Select all that apply.) A. The expectant mother often develops anemia because the fetuses have a greater demand for iron. B. Twin pregnancies come to term with the same frequency as single pregnancies. C. The mother should be counseled to increase her nutritional intake and gain more weight. D. Backache and varicose veins are often more pronounced. E. Spontaneous rupture of membranes before term is uncommon.

A, C, D (A woman with a multifetal pregnancy often develops anemia due to the increased demands of two fetuses. This should be monitored closely throughout her pregnancy. Twin pregnancies often end in prematurity. Serious efforts should be made to bring the pregnancy to term. The client may need nutrition counseling to ensure that she gains more weight than what is needed for a singleton birth. The considerable uterine distention is likely to cause backache and leg varicosities. Maternal support hose should be recommended. Spontaneous rupture of membranes before term is common.)

Which congenital malformations result from multifactorial inheritance (Select all that apply)? a. Cleft lip b. Congenital heart disease c. Cri du chat syndrome d. Anencephaly e. Pyloric stenosis

A,B,D,E

Along with gas exchange and nutrient transfer, the placenta produces many hormones necessary for normal pregnancy. These include (select all that apply) a. Human chorionic gonadotropin (hCG) b. Insulin c. Estrogen d. Progesterone e. Testosterone

A,C,D

Achieving and maintaining euglycemia comprise the primary goals of medical therapy for the pregnant woman with diabetes. These goals are achieved through a combination of diet, insulin, exercise, and blood glucose monitoring. The target blood glucose levels 1 hour after a meal should be: _________________

ANS: 130 to 140 mg/dL Target levels of blood glucose during pregnancy are lower than nonpregnant values. Accepted fasting levels are between 65 and 95 mg/dL, and 1-hour postmeal levels should be less than 130 to 140 mg/dL. Two-hour postmeal levels should be 120 mg/dL or less. PTS: 1 DIF: Cognitive Level: Application REF: 273 OBJ: Nursing Process: Implementation MSC: Client Needs: Health Promotion and Maintenance

34. A woman is 6 weeks pregnant. She has had a previous spontaneous abortion at 14 weeks of gestation and a pregnancy that ended at 38 weeks with the birth of a stillborn girl. What is her gravidity and parity using the GTPAL system? ___________________

ANS: 3-1-0-1-0 The correct calculation of this woman's gravidity and parity is 3-1-0-1-0. Using the GPTAL system, this client's gravidity and parity information is calculated as follows: G: Total number of times the woman has been pregnant (she is pregnant for the third time) T: Number of pregnancies carried to term (she has had only one pregnancy that resulted in a fetus at term) P: Number of pregnancies that resulted in a preterm birth (none) A: Abortions or miscarriages before the period of viability (she has had one) L: Number of children born who are currently living (she has no living children)

33. Spontaneous termination of a pregnancy is considered to be an abortion if: a. The pregnancy is less than 20 weeks. b. The fetus weighs less than 1000 g. c. The products of conception are passed intact. d. No evidence exists of intrauterine infection.

ANS: A An abortion is the termination of pregnancy before the age of viability (20 weeks). The weight of the fetus is not considered because some older fetuses may have a low birth weight. A spontaneous abortion may be complete or incomplete. A spontaneous abortion may be caused by many problems, one being intrauterine infection.

37. The priority nursing intervention when admitting a pregnant woman who has experienced a bleeding episode in late pregnancy is to: a. Assess fetal heart rate (FHR) and maternal vital signs b. Perform a venipuncture for hemoglobin and hematocrit levels c. Place clean disposable pads to collect any drainage d. Monitor uterine contractions

ANS: A Assessment of the FHR and maternal vital signs will assist the nurse in determining the degree of the blood loss and its effect on the mother and fetus. The most important assessment is to check mother/fetal well-being. The blood levels can be obtained later. It is important to assess future bleeding; however, the top priority remains mother/fetal well-being. Monitoring uterine contractions is important but not the top priority.

42. Approximately 10% to 15% of all clinically recognized pregnancies end in miscarriage. Which is the most common cause of spontaneous abortion? a. Chromosomal abnormalities c. Endocrine imbalance b. Infections d. Immunologic factors

ANS: A At least 50% of pregnancy losses result from chromosomal abnormalities that are incompatible with life. Maternal infection may be a cause of early miscarriage. Endocrine imbalances such as hypothyroidism or diabetes are possible causes for early pregnancy loss. Women who have repeated early pregnancy losses appear to have immunologic factors that play a role in spontaneous abortion incidents.

14. Your patient is being induced because of her worsening preeclampsia. She is also receiving magnesium sulfate. It appears that her labor has not become active despite several hours of oxytocin administration. She asks the nurse, "Why is it taking so long?" The most appropriate response by the nurse would be: a. "The magnesium is relaxing your uterus and competing with the oxytocin. It may increase the duration of your labor." b. "I don't know why it is taking so long." c. "The length of labor varies for different women." d. "Your baby is just being stubborn."

ANS: A Because magnesium sulfate is a tocolytic agent, its use may increase the duration of labor. The amount of oxytocin needed to stimulate labor may be more than that needed for the woman who is not receiving magnesium sulfate. "I don't know why it is taking so long" is not an appropriate statement for the nurse to make. Although the length of labor does vary in different women, the most likely reason this woman's labor is protracted is the tocolytic effect of magnesium sulfate. The behavior of the fetus has no bearing on the length of labor.

2. Because pregnant women may need surgery during pregnancy, nurses should be aware that: a. The diagnosis of appendicitis may be difficult because the normal signs and symptoms mimic some normal changes in pregnancy. b. Rupture of the appendix is less likely in pregnant women because of the close monitoring. c. Surgery for intestinal obstructions should be delayed as long as possible because it usually affects the pregnancy. d. When pregnancy takes over, a woman is less likely to have ovarian problems that require invasive responses.

ANS: A Both appendicitis and pregnancy are linked with nausea, vomiting, and increased white blood cell count. Rupture of the appendix is two to three times more likely in pregnant women. Surgery to remove obstructions should be done right away. It usually does not affect the pregnancy. Pregnancy predisposes a woman to ovarian problems.

39. Which order should the nurse expect for a patient admitted with a threatened abortion? a. Bed rest b. Ritodrine IV c. NPO d. Narcotic analgesia every 3 hours, prn

ANS: A Decreasing the woman's activity level may alleviate the bleeding and allow the pregnancy to continue. Ritodrine is not the first drug of choice for tocolytic medications. There is no reason for having the woman placed NPO. At times dehydration may produce contractions, so hydration is important. Narcotic analgesia will not decrease the contractions. It may mask the severity of the contractions.

10. A woman with severe preeclampsia has been receiving magnesium sulfate by intravenous infusion for 8 hours. The nurse assesses the woman and documents the following findings: temperature of 37.1° C, pulse rate of 96 beats/min, respiratory rate of 24 breaths/min, blood pressure (BP) of 155/112 mm Hg, 3+ deep tendon reflexes, and no ankle clonus. The nurse calls the physician, anticipating an order for: a. Hydralazine. c. Diazepam. b. Magnesium sulfate bolus. d. Calcium gluconate.

ANS: A Hydralazine is an antihypertensive commonly used to treat hypertension in severe preeclampsia. Typically it is administered for a systolic BP greater than 160 mm Hg or a diastolic BP greater than 110 mm Hg. An additional bolus of magnesium sulfate may be ordered for increasing signs of central nervous system irritability related to severe preeclampsia (e.g., clonus) or if eclampsia develops. Diazepam sometimes is used to stop or shorten eclamptic seizures. Calcium gluconate is used as the antidote for magnesium sulfate toxicity. The client is not currently displaying any signs or symptoms of magnesium toxicity.

19. In planning care for women with preeclampsia, nurses should be aware that: a. Induction of labor is likely, as near term as possible. b. If at home, the woman should be confined to her bed, even with mild preeclampsia. c. A special diet low in protein and salt should be initiated. d. Vaginal birth is still an option, even in severe cases.

ANS: A Induction of labor is likely, as near term as possible; however, at less than 37 weeks of gestation, immediate delivery may not be in the best interest of the fetus. Strict bed rest is becoming controversial for mild cases; some women in the hospital are even allowed to move around. Diet and fluid recommendations are much the same as for healthy pregnant women, although some authorities have suggested a diet high in protein. Women with severe preeclampsia should expect a cesarean delivery.

16. The nurse caring for pregnant women must be aware that the most common medical complication of pregnancy is: a. Hypertension. c. Hemorrhagic complications. b. Hyperemesis gravidarum. d. Infections.

ANS: A Preeclampsia and eclampsia are two noted deadly forms of hypertension. A large percentage of pregnant women will have nausea and vomiting, but a relatively few have the severe form called hyperemesis gravidarum. Hemorrhagic complications are the second most common medical complication of pregnancy; hypertension is the most common.

5. In caring for the woman with disseminated intravascular coagulation (DIC), what order should the nurse anticipate? a. Administration of blood b. Preparation of the client for invasive hemodynamic monitoring c. Restriction of intravascular fluids d. Administration of steroids

ANS: A Primary medical management in all cases of DIC involves correction of the underlying cause, volume replacement, blood component therapy, optimization of oxygenation and perfusion status, and continued reassessment of laboratory parameters. Central monitoring would not be ordered initially in a client with DIC because this can contribute to more areas of bleeding. Management of DIC would include volume replacement, not volume restriction. Steroids are not indicated for the management of DIC.

15. What nursing diagnosis would be the most appropriate for a woman experiencing severe preeclampsia? a. Risk for injury to the fetus related to uteroplacental insufficiency b. Risk for eclampsia c. Risk for deficient fluid volume related to increased sodium retention secondary to administration of MgSO4 d. Risk for increased cardiac output related to use of antihypertensive drugs

ANS: A Risk for injury to the fetus related to uteroplacental insufficiency is the most appropriate nursing diagnosis for this client scenario. Other diagnoses include Risk to fetus related to preterm birth and abruptio placentae. Eclampsia is a medical, not a nursing, diagnosis. There would be a risk for excess, not deficient, fluid volume related to increased sodium retention. There would be a risk for decreased, not increased, cardiac output related to the use of antihypertensive drugs.

4. In caring for an immediate postpartum client, you note petechiae and oozing from her IV site. You would monitor her closely for the clotting disorder: a. Disseminated intravascular coagulation (DIC) b. Amniotic fluid embolism (AFE) c. Hemorrhage d. HELLP syndrome

ANS: A The diagnosis of DIC is made according to clinical findings and laboratory markers. Physical examination reveals unusual bleeding. Petechiae may appear around a blood pressure cuff on the woman's arm. Excessive bleeding may occur from the site of slight trauma such as venipuncture sites. These symptoms are not associated with AFE, nor is AFE a bleeding disorder. Hemorrhage occurs for a variety of reasons in the postpartum client. These symptoms are associated with DIC. Hemorrhage would be a finding associated with DIC and is not a clotting disorder in and of itself. HELLP is not a clotting disorder, but it may contribute to the clotting disorder DIC.

14. A woman who has completed one pregnancy with a fetus (or fetuses) reaching the stage of fetal viability is called a: a. Primipara. b. Primigravida. c. Multipara. d. Nulligravida.

ANS: A A primipara is a woman who has completed one pregnancy with a viable fetus. To remember terms, keep in mind: gravida is a pregnant woman; para comes from parity, meaning a viable fetus; primi means first; multi means many; and null means none. A primigravida is a woman pregnant for the first time. A multipara is a woman who has completed two or more pregnancies with a viable fetus. A nulligravida is a woman who has never been pregnant.

2. A woman at 10 weeks of gestation who is seen in the prenatal clinic with presumptive signs and symptoms of pregnancy likely will have: a. Amenorrhea. b. Positive pregnancy test. c. Chadwick's sign. d. Hegar's sign.

ANS: A Amenorrhea is a presumptive sign of pregnancy. Presumptive signs of pregnancy are felt by the woman. A positive pregnancy test, the presence of Chadwick's sign, and the presence of Hegar's sign all are probable signs of pregnancy.

5. During a client's physical examination the nurse notes that the lower uterine segment is soft on palpation. The nurse would document this finding as: a. Hegar's sign b. McDonald's sign c. Chadwick's sign d. Goodell's sign

ANS: A At approximately 6 weeks of gestation, softening and compressibility of the lower uterine segment occur; this is called Hegar's sign. McDonald's sign indicates a fast food restaurant. Chadwick's sign is the blue-violet coloring of the cervix caused by increased vascularity; this occurs around the fourth week of gestation. Softening of the cervical tip is called Goodell's sign, which may be observed around the sixth week of pregnancy.

A woman is 15 weeks pregnant with her first baby. She asks how long it will be before she feels the baby move. The best answer is: a. "You should have felt the baby move by now." b. "Within the next month, you should start to feel fluttering sensations." c. "The baby is moving; however, you can't feel it yet." d. "Some babies are quiet, and you don't feel them move."

B

Marfan syndrome is an autosomal dominant genetic disorder that displays as weakness of the connective tissue, joint deformities, ocular dislocation, and weakness to the aortic wall and root. While providing care to a client with Marfan syndrome during labor, which intervention should the nurse complete first? a. Antibiotic prophylaxis b. b-Blockers c. Surgery d. Regional anesthesia

ANS: A Because of the potential for cardiac involvement during the third trimester and after birth, treatment with prophylactic antibiotics is highly recommended. b-Blockers and restricted activity are recommended as treatment modalities earlier in the pregnancy. Regional anesthesia is well tolerated by clients with Marfan syndrome; however, it is not essential to care. Adequate labor support may be all that is necessary if an epidural is not part of the womans birth plan. Surgery for cardiovascular changes such as mitral valve prolapse, aortic regurgitation, root dilation, or dissection may be necessary. Mortality rates may be as high as 50% in women who have severe cardiac disease. PTS: 1 DIF: Cognitive Level: Analysis REF: 287 OBJ: Nursing Process: Implementation MSC: Client Needs: Physiologic Integrity

As related to the care of the patient with anemia, the nurse should be aware that: a. It is the most common medical disorder of pregnancy. b. It can trigger reflex brachycardia. c. The most common form of anemia is caused by folate deficiency. d. Thalassemia is a European version of sickle cell anemia.

ANS: A Combined with any other complication, anemia can result in congestive heart failure. Reflex bradycardia is a slowing of the heart in response to the blood flow increases immediately after birth. The most common form of anemia is iron deficiency anemia. Both thalassemia and sickle cell hemoglobinopathy are hereditary but not directly related or confined to geographic areas. PTS: 1 DIF: Cognitive Level: Knowledge REF: 290 OBJ: Nursing Process: Planning MSC: Client Needs: Physiologic Integrity

When the pregnant diabetic woman experiences hypoglycemia while hospitalized, the nurse should intervene by having the patient: a. Eat six saltine crackers. b. Drink 8 oz of orange juice with 2 tsp of sugar added. c. Drink 4 oz of orange juice followed by 8 oz of milk. d. Eat hard candy or commercial glucose wafers.

ANS: A Crackers provide carbohydrates in the form of polysaccharides. Orange juice and sugar will increase the blood sugar but not provide a slow-burning carbohydrate to sustain the blood sugar. Milk is a disaccharide and orange juice is a monosaccharide. They will provide an increase in blood sugar but will not sustain the level. Hard candy or commercial glucose wafers provide only monosaccharides. PTS: 1 DIF: Cognitive Level: Application REF: 274 OBJ: Nursing Process: Implementation MSC: Client Needs: Physiologic Integrity

10. A woman is in her seventh month of pregnancy. She has been complaining of nasal congestion and occasional epistaxis. The nurse suspects that: a. This is a normal respiratory change in pregnancy caused by elevated levels of estrogen. b. This is an abnormal cardiovascular change, and the nosebleeds are an ominous sign. c. The woman is a victim of domestic violence and is being hit in the face by her partner. d. The woman has been using cocaine intranasally.

ANS: A Elevated levels of estrogen cause capillaries to become engorged in the respiratory tract. This may result in edema in the nose, larynx, trachea, and bronchi. This congestion may cause nasal stuffiness and epistaxis. Cardiovascular changes in pregnancy may cause edema in lower extremities. Determining that the woman is a victim of domestic violence and was hit in the face cannot be made on the basis of the sparse facts provided. If the woman had been hit in the face, she most likely would have additional physical findings. Determination of the use of cocaine by the woman cannot be made on the basis of the sparse facts provided.

In assessing the knowledge of a pregestational woman with type 1 diabetes concerning changing insulin needs during pregnancy, the nurse recognizes that further teaching is warranted when the client states: a. I will need to increase my insulin dosage during the first 3 months of pregnancy. b. Insulin dosage will likely need to be increased during the second and third trimesters. c. Episodes of hypoglycemia are more likely to occur during the first 3 months. d. Insulin needs should return to normal within 7 to 10 days after birth if I am bottle-feeding.

ANS: A Insulin needs are reduced in the first trimester because of increased insulin production by the pancreas and increased peripheral sensitivity to insulin. Insulin dosage will likely need to be increased during the second and third trimesters, Episodes of hypoglycemia are more likely to occur during the first 3 months, and Insulin needs should return to normal within 7 to 10 days after birth if I am bottle-feeding are accurate statements and signify that the woman has understood the teachings regarding control of her diabetes during pregnancy. PTS: 1 DIF: Cognitive Level: Application REF: 269 OBJ: Nursing Process: Evaluation MSC: Client Needs: Physiologic Integrity

With one exception, the safest pregnancy is one in which the woman is drug and alcohol free. For women addicted to opioids, ________________________ treatment is the current standard of care during pregnancy. a. Methadone maintenance b. Detoxification c. Smoking cessation d. 4 Ps Plus

ANS: A Methadone maintenance treatment (MMT) is currently considered the standard of care for pregnant women who are dependent on heroin or other narcotics. Buprenorphine is another medication approved for opioid addiction treatment that is increasingly being used during pregnancy. Opioid replacement therapy has been shown to decrease opioid and other drug use, reduce criminal activity, improve individual functioning, and decrease rates of infections such as hepatitis B and C, HIV, and other sexually transmitted infections. Detoxification is the treatment used for alcohol addiction. Pregnant women requiring withdrawal from alcohol should be admitted for inpatient management. Women are more likely to stop smoking during pregnancy than at any other time in their lives. A smoking cessation program can assist in achieving this goal. The 4 Ps Plus is a screening tool designed specifically to identify pregnant women who need in-depth assessment related to substance abuse. PTS: 1 DIF: Cognitive Level: Application REF: 298 OBJ: Nursing Process: Planning MSC: Client Needs: Psychosocial Integrity

A woman has a history of drug use and is screened for hepatitis B during the first trimester. What is an appropriate action? a. Provide a low-protein diet. b. Offer the vaccine. c. Discuss the recommendation to bottle-feed her baby. d. Practice respiratory isolation.

ANS: B A person who has a history of high risk behaviors should be offered the hepatitis B vaccine. Care is supportive and includes bed rest and a high-protein, low-fat diet. The first trimester is too early to discuss feeding methods with a woman in the high risk category. Hepatitis B is transmitted through blood. PTS: 1 DIF: Cognitive Level: Application REF: 298 OBJ: Nursing Process: Planning MSC: Client Needs: Health Promotion and Maintenance

Screening at 24 weeks of gestation reveals that a pregnant woman has gestational diabetes mellitus (GDM). In planning her care, the nurse and the woman mutually agree that an expected outcome is to prevent injury to the fetus as a result of GDM. The nurse identifies that the fetus is at greatest risk for: a. Macrosomia. b. Congenital anomalies of the central nervous system. c. Preterm birth. d. Low birth weight.

ANS: A Poor glycemic control later in pregnancy increases the rate of fetal macrosomia. Poor glycemic control during the preconception time frame and into the early weeks of the pregnancy is associated with congenital anomalies. Preterm labor or birth is more likely to occur with severe diabetes and is the greatest risk in women with pregestational diabetes. Increased weight, or macrosomia, is the greatest risk factor for this woman. PTS: 1 DIF: Cognitive Level: Comprehension REF: 270 OBJ: Nursing Process: Planning, Implementation MSC: Client Needs: Physiologic Integrity

With regard to the association of maternal diabetes and other risk situations affecting mother and fetus, nurses should be aware that: a. Diabetic ketoacidosis (DKA) can lead to fetal death at any time during pregnancy. b. Hydramnios occurs approximately twice as often in diabetic pregnancies. c. Infections occur about as often and are considered about as serious in diabetic and nondiabetic pregnancies. d. Even mild to moderate hypoglycemic episodes can have significant effects on fetal well-being.

ANS: A Prompt treatment of DKA is necessary to save the fetus and the mother. Hydramnios occurs 10 times more often in diabetic pregnancies. Infections are more common and more serious in pregnant women with diabetes. Mild to moderate hypoglycemic episodes do not appear to have significant effects on fetal well-being. PTS: 1 DIF: Cognitive Level: Comprehension REF: 270 OBJ: Nursing Process: Planning MSC: Client Needs: Physiologic Integrity

Prophylaxis of subacute bacterial endocarditis is given before and after birth when a pregnant woman has: a. Valvular disease. b. Congestive heart disease. c. Arrhythmias. d. Postmyocardial infarction.

ANS: A Prophylaxis for intrapartum endocarditis and pulmonary infection may be provided for women who have mitral valve stenosis. Prophylaxis for intrapartum endocarditis is not indicated for congestive heart disease, arrhythmias, or after myocardial infarction. PTS: 1 DIF: Cognitive Level: Comprehension REF: 285 OBJ: Nursing Process: Implementation MSC: Client Needs: Physiologic Integrity

13. Appendicitis may be difficult to diagnose in pregnancy because the appendix is: a. Displaced upward and laterally, high and to the right. b. Displaced upward and laterally, high and to the left. c. Deep at McBurney point. d. Displaced downward and laterally, low and to the right.

ANS: A The appendix is displaced high and to the right, beyond McBurney point.

Which major neonatal complication is carefully monitored after the birth of the infant of a diabetic mother? a. Hypoglycemia b. Hypercalcemia c. Hypobilirubinemia d. Hypoinsulinemia

ANS: A The neonate is at highest risk for hypoglycemia because fetal insulin production is accelerated during pregnancy to metabolize excessive glucose from the mother. At birth, the maternal glucose supply stops and the neonatal insulin exceeds the available glucose, thus leading to hypoglycemia. Hypocalcemia is associated with preterm birth, birth trauma, and asphyxia, all common problems of the infant of a diabetic mother. Excess erythrocytes are broken down after birth and release large amounts of bilirubin into the neonates circulation, with resulting hyperbilirubinemia. Because fetal insulin production is accelerated during pregnancy, the neonate presents with hyperinsulinemia. PTS: 1 DIF: Cognitive Level: Comprehension REF: 272 OBJ: Nursing Process: Planning MSC: Client Needs: Health Promotion and Maintenance

19. The mucous plug that forms in the endocervical canal is called the: a. Operculum. b. Leukorrhea. c. Funic souffle. d. Ballottement.

ANS: A The operculum protects against bacterial invasion. Leukorrhea is the mucus that forms the endocervical plug (the operculum). The funic souffle is the sound of blood flow

26. A first-time mother at 18 weeks of gestation comes for her regularly scheduled prenatal visit. The client tells the nurse that she is afraid that she is going into premature labor because she is beginning to have regular contractions. The nurse explains that this is the Braxton Hicks sign and teaches the client that this type of contraction: a. Is painless. b. Increases with walking. c. Causes cervical dilation. d. Impedes oxygen flow to the fetus.

ANS: A Uterine contractions can be felt through the abdominal wall soon after the fourth month of gestation. Braxton Hicks contractions are regular and painless and continue throughout the pregnancy. Although they are not painful, some women complain that they are annoying. Braxton Hicks contractions usually cease with walking or exercise. They can be mistaken for true labor; however, they do not increase in intensity or frequency or cause cervical dilation. In addition, they facilitate uterine blood flow through the intervillous spaces of the placenta and promote oxygen delivery to the fetus.

32. A woman is in for a routine prenatal checkup. You are assessing her urine for proteinuria. Which findings are considered normal (Select all that apply)? a. Dipstick assessment of trace to +1 b. <300 mg/24 hours c. Dipstick assessment of +2 d. >300 mg/24 hours

ANS: A, B Small amounts of protein in the urine are acceptable during pregnancy. The presence of protein in greater amounts may indicate renal problems. A dipstick assessment of +2 and >300 mg/24 hours are excessive amounts of protein in the urine and should be evaluated further.

Congenital anomalies can occur with the use of antiepileptic drugs (AEDs), including (Select all that apply): a. Cleft lip. b. Congenital heart disease. c. Neural tube defects. d. Gastroschisis. e. Diaphragmatic hernia.

ANS: A, B, C Congenital anomalies that can occur with AEDs include cleft lip or palate, congenital heart disease, urogenital defects, and neural tube defects. Gastroschisis and diaphragmatic hernia are not associated with the use of AEDs. PTS: 1 DIF: Cognitive Level: Comprehension REF: 294 OBJ: Nursing Process: Planning MSC: Client Needs: Physiologic Integrity

29. Physiologic anemia often occurs during pregnancy as a result of: a. Inadequate intake of iron. b. Dilution of hemoglobin concentration. c. The fetus establishing iron stores. d. Decreased production of erythrocytes.

ANS: B When blood volume expansion is more pronounced and occurs earlier than the increase in red blood cells, the woman has physiologic anemia, which is the result of dilution of hemoglobin concentration rather than inadequate hemoglobin. Inadequate intake of iron may lead to true anemia. There is an increased production of erythrocytes during pregnancy.

45. The reported incidence of ectopic pregnancy in the United States has risen steadily over the past 2 decades. Causes include the increase in STDs accompanied by tubal infection and damage. The popularity of contraceptive devices such as the IUD has also increased the risk for ectopic pregnancy. The nurse who suspects that a patient has early signs of ectopic pregnancy should be observing her for symptoms such as (Select all that apply): a. Pelvic pain b. Abdominal pain c. Unanticipated heavy bleeding d. Vaginal spotting or light bleeding e. Missed period

ANS: A, B, D, E A missed period or spotting can easily be mistaken by the patient as early signs of pregnancy. More subtle signs depend on exactly where the implantation occurs. The nurse must be thorough in her assessment because pain is not a normal symptom of early pregnancy. As the fallopian tube tears open and the embryo is expelled, the patient often exhibits severe pain accompanied by intraabdominal hemorrhage. This may progress to hypovolemic shock with minimal or even no external bleeding. In about half of women, shoulder and neck pain results from irritation of the diaphragm from the hemorrhage.

Diabetes refers to a group of metabolic diseases characterized by hyperglycemia resulting from defects in insulin action, insulin secretion, or both. Over time, diabetes causes significant changes in the microvascular and macrovascular circulations. These complications include: a. Atherosclerosis. b. Retinopathy. c. IUFD. d. Nephropathy. e. Autonomcs neuropathy.

ANS: A, B, D, E These structural changes are most likely to affect a variety of systems, including the heart, eyes, kidneys, and nerves. Intrauterine fetal death (stillbirth) remains a major complication of diabetes in pregnancy; however, this is a fetal complication. PTS: 1 DIF: Cognitive Level: Comprehension REF: 268 OBJ: Nursing Process: Diagnosis MSC: Client Needs: Physiologic Integrity

31. The diagnosis of pregnancy is based on which positive signs of pregnancy (Select all that apply)? a. Identification of fetal heartbeat b. Palpation of fetal outline c. Visualization of the fetus d. Verification of fetal movement e. Positive hCG test

ANS: A, C, D Identification of fetal heartbeat, visualization of the fetus, and verification of fetal movement all are positive, objective signs of pregnancy. Palpation of fetal outline and a positive hCG test are probable signs of pregnancy. A tumor also can be palpated. Medication and tumors may lead to false-positive results on pregnancy tests.

44. A client who has undergone a dilation and curettage for early pregnancy loss is likely to be discharged the same day. The nurse must ensure that vital signs are stable, bleeding has been controlled, and the woman has adequately recovered from the administration of anesthesia. To promote an optimal recovery, discharge teaching should include (Select all that apply): a. Iron supplementation. b. Resumption of intercourse at 6 weeks following the procedure. c. Referral to a support group if necessary. d. Expectation of heavy bleeding for at least 2 weeks. e. Emphasizing the need for rest.

ANS: A, C, E The woman should be advised to consume a diet high in iron and protein. For many women iron supplementation also is necessary. Acknowledge that the client has experienced a loss, albeit early. She can be taught to expect mood swings and possibly depression. Referral to a support group, clergy, or professional counseling may be necessary. Discharge teaching should emphasize the need for rest. Nothing should be placed in the vagina for 2 weeks after the procedure. This includes tampons and vaginal intercourse. The purpose of this recommendation is to prevent infection. Should infection occur, antibiotics may be prescribed. The client should expect a scant, dark discharge for 1 to 2 weeks. Should heavy, profuse, or bright bleeding occur, she should be instructed to contact her provider.

12. The patient that you are caring for has severe preeclampsia and is receiving a magnesium sulfate infusion. You become concerned after assessment when the woman exhibits: a. A sleepy, sedated affect. c. Deep tendon reflexes of 2. b. A respiratory rate of 10 breaths/min. d. Absent ankle clonus.

ANS: B A respiratory rate of 10 breaths/min indicates that the client is experiencing respiratory depression from magnesium toxicity. Because magnesium sulfate is a central nervous system depressant, the client will most likely become sedated when the infusion is initiated. Deep tendon reflexes of 2 and absent ankle clonus are normal findings.

3. What laboratory marker is indicative of disseminated intravascular coagulation (DIC)? a. Bleeding time of 10 minutes c. Thrombocytopenia b. Presence of fibrin split products d. Hyperfibrinogenemia

ANS: B Degradation of fibrin leads to the accumulation of fibrin split products in the blood. Bleeding time in DIC is normal. Low platelets may occur with but are not indicative of DIC because they may result from other coagulopathies. Hypofibrinogenemia would occur with DIC.

38. A patient with pregnancy-induced hypertension is admitted complaining of pounding headache, visual changes, and epigastric pain. Nursing care is based on the knowledge that these signs are an indication of: a. Anxiety due to hospitalization. b. Worsening disease and impending convulsion. c. Effects of magnesium sulfate. d. Gastrointestinal upset.

ANS: B Headache and visual disturbances are caused by increased cerebral edema. Epigastric pain indicates distention of the hepatic capsules and often warns that a convulsion is imminent. These are danger signs showing increased cerebral edema and impending convulsion and should be treated immediately. The patient has not been started on magnesium sulfate treatment yet. Also, these are not anticipated effects of the medication.

32. Which maternal condition always necessitates delivery by cesarean section? a. Partial abruptio placentae c. Ectopic pregnancy b. Total placenta previa d. Eclampsia

ANS: B In total placenta previa, the placenta completely covers the cervical os. The fetus would die if a vaginal delivery occurred. If the mother has stable vital signs and the fetus is alive, a vaginal delivery can be attempted in cases of partial abruptio placentae. If the fetus has died, a vaginal delivery is preferred. The most common ectopic pregnancy is a tubal pregnancy, which is usually detected and treated in the first trimester. Labor can be safely induced if the eclampsia is under control.

30. Which condition would not be classified as a bleeding disorder in late pregnancy? a. Placenta previa. c. Spontaneous abortion. b. Abruptio placentae. d. Cord insertion.

ANS: C Spontaneous abortion is another name for miscarriage; by definition it occurs early in pregnancy. Placenta previa is a cause of bleeding disorders in later pregnancy. Abruptio placentae is a cause of bleeding disorders in later pregnancy. Cord insertion is a cause of bleeding disorders in later pregnancy.

43. The nurse caring for a woman hospitalized for hyperemesis gravidarum should expect that initial treatment to involve: a. Corticosteroids to reduce inflammation. b. IV therapy to correct fluid and electrolyte imbalances. c. An antiemetic, such as pyridoxine, to control nausea and vomiting. d. Enteral nutrition to correct nutritional deficits.

ANS: B Initially, the woman who is unable to keep down clear liquids by mouth requires IV therapy for correction of fluid and electrolyte imbalances. Corticosteroids have been used successfully to treat refractory hyperemesis gravidarum; however, they are not the expected initial treatment for this disorder. Pyridoxine is vitamin B6, not an antiemetic. Promethazine, a common antiemetic, may be prescribed. In severe cases of hyperemesis gravidarum, enteral nutrition via a feeding tube may be necessary to correct maternal nutritional deprivation. This is not an initial treatment for this patient.

34. An abortion in which the fetus dies but is retained within the uterus is called a(n): a. Inevitable abortion c. Incomplete abortion b. Missed abortion d. Threatened abortion

ANS: B Missed abortion refers to retention of a dead fetus in the uterus. An inevitable abortion means that the cervix is dilating with the contractions. An incomplete abortion means that not all of the products of conception were expelled. With a threatened abortion the woman has cramping and bleeding but not cervical dilation.

24. The most prevalent clinical manifestation of abruptio placentae (as opposed to placenta previa) is: a. Bleeding. c. Uterine activity. b. Intense abdominal pain. d. Cramping.

ANS: B Pain is absent with placenta previa and may be agonizing with abruptio placentae. Bleeding may be present in varying degrees for both placental conditions. Uterine activity and cramping may be present with both placental conditions.

26. A 26-year-old pregnant woman, gravida 2, para 1-0-0-1 is 28 weeks pregnant when she experiences bright red, painless vaginal bleeding. On her arrival at the hospital, what would be an expected diagnostic procedure? a. Amniocentesis for fetal lung maturity b. Ultrasound for placental location c. Contraction stress test (CST) d. Internal fetal monitoring

ANS: B The presence of painless bleeding should always alert the health care team to the possibility of placenta previa. This can be confirmed through ultrasonography. Amniocentesis would not be performed on a woman who is experiencing bleeding. In the event of an imminent delivery, the fetus would be presumed to have immature lungs at this gestational age, and the mother would be given corticosteroids to aid in fetal lung maturity. A CST would not be performed at a preterm gestational age. Furthermore, bleeding would be a contraindication to this test. Internal fetal monitoring would be contraindicated in the presence of bleeding.

23. The perinatal nurse is giving discharge instructions to a woman after suction curettage secondary to a hydatidiform mole. The woman asks why she must take oral contraceptives for the next 12 months. The best response from the nurse would be: a. "If you get pregnant within 1 year, the chance of a successful pregnancy is very small. Therefore, if you desire a future pregnancy, it would be better for you to use the most reliable method of contraception available." b. "The major risk to you after a molar pregnancy is a type of cancer that can be diagnosed only by measuring the same hormone that your body produces during pregnancy. If you were to get pregnant, it would make the diagnosis of this cancer more difficult." c. "If you can avoid a pregnancy for the next year, the chance of developing a second molar pregnancy is rare. Therefore, to improve your chance of a successful pregnancy, it is better not to get pregnant at this time." d. "Oral contraceptives are the only form of birth control that will prevent a recurrence of a molar pregnancy."

ANS: B This is an accurate statement. b-Human chorionic gonadotropin (hCG) levels will be drawn for 1 year to ensure that the mole is completely gone. There is an increased chance of developing choriocarcinoma after the development of a hydatidiform mole. The goal is to achieve a "zero" hCG level. If the woman were to become pregnant, it could obscure the presence of the potentially carcinogenic cells. Women should be instructed to use birth control for 1 year after treatment for a hydatidiform mole. The rationale for avoiding pregnancy for 1 year is to ensure that carcinogenic cells are not present. Any contraceptive method except an intrauterine device is acceptable.

27. A laboring woman with no known risk factors suddenly experiences spontaneous rupture of membranes (ROM). The fluid consists of bright red blood. Her contractions are consistent with her current stage of labor. There is no change in uterine resting tone. The fetal heart rate begins to decline rapidly after the ROM. The nurse should suspect the possibility of: a. Placenta previa. b. Vasa previa. c. Severe abruptio placentae. d. Disseminated intravascular coagulation (DIC).

ANS: B Vasa previa is the result of a velamentous insertion of the umbilical cord. The umbilical vessels are not surrounded by Wharton jelly and have no supportive tissue. They are at risk for laceration at any time, but laceration occurs most frequently during ROM. The sudden appearance of bright red blood at the time of ROM and a sudden change in the fetal heart rate without other known risk factors should immediately alert the nurse to the possibility of vasa previa. The presence of placenta previa most likely would be ascertained before labor and would be considered a risk factor for this pregnancy. In addition, if the woman had a placenta previa, it is unlikely that she would be allowed to pursue labor and a vaginal birth. With the presence of severe abruptio placentae, the uterine tonicity would typically be tetanus (i.e., a boardlike uterus). DIC is a pathologic form of diffuse clotting that consumes large amounts of clotting factors and causes widespread external bleeding, internal bleeding, or both. DIC is always a secondary diagnosis, often associated with obstetric risk factors such as HELLP syndrome. This woman did not have any prior risk factors.

1. Women with hyperemesis gravidarum: a. Are a majority, because 80% of all pregnant women suffer from it at some time. b. Have vomiting severe and persistent enough to cause weight loss, dehydration, and electrolyte imbalance. c. Need intravenous (IV) fluid and nutrition for most of their pregnancy. d. Often inspire similar, milder symptoms in their male partners and mothers.

ANS: B Women with hyperemesis gravidarum have severe vomiting; however, treatment for several days sets things right in most cases. Although 80% of pregnant women experience nausea and vomiting, fewer than 1% (0.5%) proceed to this severe level. IV administration may be used at first to restore fluid levels, but it is seldom needed for very long. Women suffering from this condition want sympathy because some authorities believe that difficult relationships with mothers and/or partners may be the cause.

Concerning the use and abuse of legal drugs or substances, nurses should be aware that: a. Although cigarette smoking causes a number of health problems, it has little direct effect on maternity-related health. b. Caucasian women are more likely to experience alcohol-related problems. c. Coffee is a stimulant that can interrupt body functions and has been related to birth defects. d. Prescription psychotherapeutic drugs taken by the mother do not affect the fetus; otherwise, they would not have been prescribed.

ANS: B African-American and poor women are more likely to use illicit substances, particularly cocaine, whereas Caucasian and educated women are more likely to use alcohol. Cigarette smoking impairs fertility and is a cause of low birth weight. Caffeine consumption has not been related to birth defects. Psychotherapeutic drugs have some effect on the fetus, and that risk must be weighed against their benefit to the mother. PTS: 1 DIF: Cognitive Level: Knowledge REF: 297 OBJ: Nursing Process: Assessment MSC: Client Needs: Psychosocial Integrity

8. The musculoskeletal system adapts to the changes that occur during pregnancy. A woman can expect to experience what change? a. Her center of gravity will shift backward. b. She will have increased lordosis. c. She will have increased abdominal muscle tone. d. She will notice decreased mobility of her pelvic joints.

ANS: B An increase in the normal lumbosacral curve (lordosis) develops, and a compensatory curvature in the cervicodorsal region develops to help the woman maintain her balance. The center of gravity shifts forward. She will have decreased muscle tone. She will notice increased mobility of her pelvic joints.

9. A 31-year-old woman believes that she may be pregnant. She took an OTC pregnancy test 1 week ago after missing her period; the test was positive. During her assessment interview, the nurse inquires about the woman's last menstrual period and asks whether she is taking any medications. The woman states that she takes medicine for epilepsy. She has been under considerable stress lately at work and has not been sleeping well. She also has a history of irregular periods. Her physical examination does not indicate that she is pregnant. She has an ultrasound scan, which reveals that she is not pregnant. What is the most likely cause of the false-positive pregnancy test result? a. She took the pregnancy test too early. b. She takes anticonvulsants. c. She has a fibroid tumor. d. She has been under considerable stress and has a hormone imbalance.

ANS: B Anticonvulsants may cause false-positive pregnancy test results. OTC pregnancy tests use enzyme-linked immunosorbent assay technology, which can yield positive results 4 days after implantation. Implantation occurs 6 to 10 days after conception. If the woman were pregnant, she would be into her third week at this point (having missed her period 1 week ago). Fibroid tumors do not produce hormones and have no bearing on hCG pregnancy tests. Although stress may interrupt normal hormone cycles (menstrual cycles), it does not affect human chorionic gonadotropin levels or produce positive pregnancy test results.

To reassure and educate pregnant clients about changes in their cardiovascular system, maternity nurses should be aware that: a. A pregnant woman experiencing disturbed cardiac rhythm, such as sinus arrhythmia requires close medical and obstetric observation, no matter how healthy she otherwise may appear. b. Changes in heart size and position and increases in blood volume create auditory changes from 20 weeks to term. c. Palpitations are twice as likely to occur in twin gestations. d. All of the above changes likely will occur.

ANS: B Auscultatory changes should be discernible after 20 weeks of gestation. A healthy woman with no underlying heart disease does not need any therapy. The maternal heart rate increases in the third trimester, but palpitations may not occur. Auditory changes are discernible at 20 weeks.

While providing care in an obstetric setting, the nurse should understand that postpartum care of the woman with cardiac disease: a. Is the same as that for any pregnant woman. b. Includes rest, stool softeners, and monitoring of the effect of activity. c. Includes ambulating frequently, alternating with active range of motion. d. Includes limiting visits with the infant to once per day.

ANS: B Bed rest may be ordered, with or without bathroom privileges. Bowel movements without stress or strain for the woman are promoted with stool softeners, diet, and fluid. Care of the woman with cardiac disease in the postpartum period is tailored to the womans functional capacity. The woman will be on bed rest to conserve energy and reduce the strain on the heart. Although the woman may need help caring for the infant, breastfeeding and infant visits are not contraindicated. PTS: 1 DIF: Cognitive Level: Comprehension REF: 290 OBJ: Nursing Process: Planning MSC: Client Needs: Physiologic Integrity

An 18-year-old client who has reached 16 weeks of gestation was recently diagnosed with pregestational diabetes. She attends her centering appointment accompanied by one of her girlfriends. This young woman appears more concerned about how her pregnancy will affect her social life than about her recent diagnosis of diabetes. Several nursing diagnoses are applicable to assist in planning adequate care. The most appropriate diagnosis at this time is: a. Risk for injury to the fetus related to birth trauma. b. Noncompliance related to lack of understanding of diabetes and pregnancy and requirements of the treatment plan. c. Deficient knowledge related to insulin administration. d. Risk for injury to the mother related to hypoglycemia or hyperglycemia.

ANS: B Before a treatment plan is developed or goals for the outcome of care are outlined, this client must come to an understanding of diabetes and the potential effects on her pregnancy. She appears to have greater concern for changes to her social life than adoption of a new self-care regimen. Risk for injury to the fetus related to either placental insufficiency or birth trauma may come much later in the pregnancy. At this time the client is having difficulty acknowledging the adjustments that she needs to make to her lifestyle to care for herself during pregnancy. The client may not yet be on insulin. Insulin requirements increase with gestation. The importance of glycemic control must be part of health teaching for this client. However, she has not yet acknowledged that changes to her lifestyle need to be made, and she may not participate in the plan of care until understanding takes place. PTS: 1 DIF: Cognitive Level: Analysis REF: 269 OBJ: Nursing Process: Diagnosis MSC: Client Needs: Psychosocial Integrity

6. Cardiovascular system changes occur during pregnancy. Which finding would be considered normal for a woman in her second trimester? a. Less audible heart sounds (S1, S2) b. Increased pulse rate c. Increased blood pressure d. Decreased red blood cell (RBC) production

ANS: B Between 14 and 20 weeks of gestation, the pulse increases about 10 to 15 beats/min, which persists to term. Splitting of S1 and S2 is more audible. In the first trimester, blood pressure usually remains the same as at the prepregnancy level, but it gradually decreases up to about 20 weeks of gestation. During the second trimester, both the systolic and the diastolic pressures decrease by about 5 to 10 mm Hg. Production of RBCs accelerates during pregnancy.

Nurses caring for antepartum women with cardiac conditions should be aware that: a. Stress on the heart is greatest in the first trimester and the last 2 weeks before labor. b. Women with class II cardiac disease should avoid heavy exertion and any activity that causes even minor symptoms. c. Women with class III cardiac disease should have 8 to 10 hours of sleep every day and limit housework, shopping, and exercise. d. Women with class I cardiac disease need bed rest through most of the pregnancy and face the possibility of hospitalization near term.

ANS: B Class II cardiac disease is symptomatic with ordinary activity. Women in this category need to avoid heavy exertion and limit regular activities as symptoms dictate. Stress is greatest between weeks 28 and 32, when homodynamic changes reach their maximum. Class III cardiac disease is symptomatic with less than ordinary activity. These women need bed rest most of the day and face the possibility of hospitalization near term. Class I cardiac disease is asymptomatic at normal levels of activity. These women can carry on limited normal activities with discretion, although they still need a good amount of sleep. PTS: 1 DIF: Cognitive Level: Comprehension REF: 284 OBJ: Nursing Process: Planning MSC: Client Needs: Physiologic Integrity

Diabetes in pregnancy puts the fetus at risk in several ways. Nurses should be aware that: a. With good control of maternal glucose levels, sudden and unexplained stillbirth is no longer a major concern. b. The most important cause of perinatal loss in diabetic pregnancy is congenital malformations. c. Infants of mothers with diabetes have the same risks for respiratory distress syndrome because of the careful monitoring. d. At birth the neonate of a diabetic mother is no longer in any risk.

ANS: B Congenital malformations account for 30% to 50% of perinatal deaths. Even with good control, sudden and unexplained stillbirth remains a major concern. Infants of diabetic mothers are at increased risk for respiratory distress syndrome. The transition to extrauterine life often is marked by hypoglycemia and other metabolic abnormalities. PTS: 1 DIF: Cognitive Level: Comprehension REF: 271 OBJ: Nursing Process: Diagnosis MSC: Client Needs: Physiologic Integrity

24. To reassure and educate pregnant clients about the functioning of their kidneys in eliminating waste products, maternity nurses should be aware that: a. Increased urinary output makes pregnant women less susceptible to urinary infection. b. Increased bladder sensitivity and then compression of the bladder by the enlarging uterus results in the urge to urinate even if the bladder is almost empty. c. Renal (kidney) function is more efficient when the woman assumes a supine position. d. Using diuretics during pregnancy can help keep kidney function regular.

ANS: B First bladder sensitivity and then compression of the bladder by the uterus result in the urge to urinate more often. Numerous anatomic changes make a pregnant woman more susceptible to urinary tract infection. Renal function is more efficient when the woman lies in the lateral recumbent position and less efficient when she is supine. Diuretic use during pregnancy can overstress the system and cause problems.

A woman is at 14 weeks of gestation. The nurse would expect to palpate the fundus at which level? a. Not palpable above the symphysis at this time b. Slightly above the symphysis pubis c. At the level of the umbilicus d. Slightly above the umbilicus

ANS: B In normal pregnancies, the uterus grows at a predictable rate. It may be palpated above the symphysis pubis sometime between the twelfth and fourteenth weeks of pregnancy. As the uterus grows, it may be palpated above the symphysis pubis sometime between the twelfth and fourteenth weeks of pregnancy. The uterus rises gradually to the level of the umbilicus at 22 to 24 weeks of gestation.

The use of methamphetamine (meth) has been described as a significant drug problem in the United States. In order to provide adequate nursing care to this client population the nurse must be cognizant that methamphetamine: a. Is similar to opiates. b. Is a stimulant with vasoconstrictive characteristics. c. Should not be discontinued during pregnancy. d. Is associated with a low rate of relapse.

ANS: B Methamphetamines are stimulants with vasoconstrictive characteristics similar to cocaine and are used similarly. As is the case with cocaine users, methamphetamine users are urged to immediately stop all use during pregnancy. Unfortunately, because methamphetamine users are extremely psychologically addicted, the rate of relapse is very high. PTS: 1 DIF: Cognitive Level: Comprehension REF: 299 OBJ: Nursing Process: Assessment MSC: Client Needs: Psychosocial Integrity

30. A patient in her first trimester complains of nausea and vomiting. She asks, "Why does this happen?" The nurse's best response is: a. "It is due to an increase in gastric motility." b. "It may be due to changes in hormones." c. "It is related to an increase in glucose levels." d. "It is caused by a decrease in gastric secretions."

ANS: B Nausea and vomiting are believed to be caused by increased levels of hormones, decreased gastric motility, and hypoglycemia. Gastric motility decreases during pregnancy. Glucose levels decrease in the first trimester. Although gastric secretions decrease, this is not the main cause of nausea and vomiting.

A new mother with which of these thyroid disorders would be strongly discouraged from breastfeeding? a. Hyperthyroidism b. Phenylketonuria (PKU) c. Hypothyroidism d. Thyroid storm

ANS: B PKU is a cause of mental retardation in infants; mothers with PKU pass on phenylalanine. A woman with hyperthyroidism or hypothyroidism would have no particular reason not to breastfeed. A thyroid storm is a complication of hyperthyroidism. PTS: 1 DIF: Cognitive Level: Comprehension REF: 283 OBJ: Nursing Process: Planning MSC: Client Needs: Physiologic Integrity

3. The nurse teaches a pregnant woman about the presumptive, probable, and positive signs of pregnancy. The woman demonstrates understanding of the nurse's instructions if she states that a positive sign of pregnancy is: a. A positive pregnancy test. b. Fetal movement palpated by the nurse-midwife. c. Braxton Hicks contractions. d. Quickening.

ANS: B Positive signs of pregnancy are attributed to the presence of a fetus, such as hearing the fetal heartbeat or palpating fetal movement. A positive pregnancy test and Braxton Hicks contractions are probable signs of pregnancy. Quickening is a presumptive sign of pregnancy.

17. Nurses should be aware that HELLP syndrome: a. Is a mild form of preeclampsia. b. Can be diagnosed by a nurse alert to its symptoms. c. Is characterized by hemolysis, elevated liver enzymes, and low platelets. d. Is associated with preterm labor but not perinatal mortality.

ANS: C The acronym HELLP stands for hemolysis (H), elevated liver enzymes (EL), and low platelets (LP). HELLP syndrome is a variant of severe preeclampsia. HELLP syndrome is difficult to identify because the symptoms often are not obvious. It must be diagnosed in the laboratory. Preterm labor is greatly increased, and so is perinatal mortality.

Preconception counseling is critical to the outcome of diabetic pregnancies because poor glycemic control before and during early pregnancy is associated with: a. Frequent episodes of maternal hypoglycemia. b. Congenital anomalies in the fetus. c. Polyhydramnios. d. Hyperemesis gravidarum.

ANS: B Preconception counseling is particularly important because strict metabolic control before conception and in the early weeks of gestation is instrumental in decreasing the risks of congenital anomalies. Frequent episodes of maternal hypoglycemia may occur during the first trimester (not before conception) as a result of hormone changes and the effects on insulin production and usage. Hydramnios occurs about 10 times more often in diabetic pregnancies than in nondiabetic pregnancies. Typically it is seen in the third trimester of pregnancy. Hyperemesis gravidarum may exacerbate hypoglycemic events because the decreased food intake by the mother and glucose transfer to the fetus contribute to hypoglycemia. PTS: 1 DIF: Cognitive Level: Comprehension REF: 270 OBJ: Nursing Process: Planning MSC: Client Needs: Physiologic Integrity

Which finding in the urine analysis of a pregnant woman is considered a variation of normal? a. Proteinuria b. Glycosuria c. Bacteria in the urine. d. Ketones in the urine.

ANS: B Small amounts of glucose may indicate "physiologic spilling." The presence of protein could indicate kidney disease or preeclampsia. Urinary tract infections are associated with bacteria in the urine. An increase in ketones indicates that the patient is exercising too strenuously or has an inadequate fluid and food intake

15. Which time-based description of a stage of development in pregnancy is accurate? a. Viability—22 to 37 weeks since the last menstrual period (LMP) (assuming a fetal weight >500 g) b. Term—pregnancy from the beginning of week 38 of gestation to the end of week 42 c. Preterm—pregnancy from 20 to 28 weeks d. Postdate—pregnancy that extends beyond 38 weeks

ANS: B Term is 38 to 42 weeks of gestation. Viability is the ability of the fetus to live outside the uterus before coming to term, or 22 to 24 weeks since LMP. Preterm is 20 to 37 weeks of gestation. Postdate or postterm is a pregnancy that extends beyond 42 weeks or what is considered the limit of full term.

1. A woman's obstetric history indicates that she is pregnant for the fourth time and all of her children from previous pregnancies are living. One was born at 39 weeks of gestation, twins were born at 34 weeks of gestation, and another child was born at 35 weeks of gestation. What is her gravidity and parity using the GTPAL system? a. 3-1-1-1-3 b. 4-1-2-0-4 c. 3-0-3-0-3 d. 4-2-1-0-3

ANS: B The correct calculation of this woman's gravidity and parity is 4-1-2-0-4. The numbers reflect the woman's gravidity and parity information. Using the GPTAL system, her information is calculated as: G: The first number reflects the total number of times the woman has been pregnant; she is pregnant for the fourth time. T: This number indicates the number of pregnancies carried to term, not the number of deliveries at term; only one of her pregnancies has resulted in a fetus at term. P: This is the number of pregnancies that resulted in a preterm birth; the woman has had two pregnancies in which she delivered preterm. A: This number signifies whether the woman has had any abortions or miscarriages before the period of viability; she has not. L: This number signifies the number of children born that currently are living; the woman has four children.

28. The maternity nurse understands that vascular volume increases 40% to 60% during pregnancy to: a. Compensate for decreased renal plasma flow. b. Provide adequate perfusion of the placenta. c. Eliminate metabolic wastes of the mother. d. Prevent maternal and fetal dehydration.

ANS: B The primary function of increased vascular volume is to transport oxygen and nutrients to the fetus via the placenta. Renal plasma flow increases during pregnancy. Assisting with pulling metabolic wastes from the fetus for maternal excretion is one purpose of the increased vascular volume.

17. To reassure and educate pregnant clients about changes in the uterus, nurses should be aware that: a. Lightening occurs near the end of the second trimester as the uterus rises into a different position. b. The woman's increased urinary frequency in the first trimester is the result of exaggerated uterine antireflexion caused by softening. c. Braxton Hicks contractions become more painful in the third trimester, particularly if the woman tries to exercise. d. The uterine souffle is the movement of the fetus.

ANS: B The softening of the lower uterine segment is called Hegar's sign. Lightening occurs in the last 2 weeks of pregnancy, when the fetus descends. Braxton Hicks contractions become more defined in the final trimester but are not painful. Walking or exercise usually causes them to stop. The uterine souffle is the sound made by blood in the uterine arteries; it can be heard with a fetal stethoscope.

A 26-year-old primigravida has come to the clinic for her regular prenatal visit at 12 weeks. She appears thin and somewhat nervous. She reports that she eats a well-balanced diet, although her weight is 5 pounds less than it was at her last visit. The results of laboratory studies confirm that she has a hyperthyroid condition. Based on the available data, the nurse formulates a plan of care. What nursing diagnosis is most appropriate for the woman at this time? a. Deficient fluid volume b. Imbalanced nutrition: less than body requirements c. Imbalanced nutrition: more than body requirements d. Disturbed sleep pattern

ANS: B This clients clinical cues include weight loss, which would support the nursing diagnosis of Imbalanced nutrition: less than body requirements. No clinical signs or symptoms support the nursing diagnosis of Deficient fluid volume. This client reports weight loss, not weight gain. Imbalanced nutrition: more than body requirements is not an appropriate nursing diagnosis. Although the client reports nervousness, based on the clients other clinical symptoms the most appropriate nursing diagnosis would be Imbalanced nutrition: less than body requirements. PTS: 1 DIF: Cognitive Level: Analysis REF: 282 OBJ: Nursing Process: Diagnosis MSC: Client Needs: Physiologic Integrity

In terms of the incidence and classification of diabetes, maternity nurses should know that: a. Type 1 diabetes is most common. b. Type 2 diabetes often goes undiagnosed. c. Gestational diabetes mellitus (GDM) means that the woman will be receiving insulin treatment until 6 weeks after birth. d. Type 1 diabetes may become type 2 during pregnancy.

ANS: B Type 2 diabetes often goes undiagnosed because hyperglycemia develops gradually and often is not severe. Type 2 diabetes, sometimes called adult onset diabetes, is the most common. GDM refers to any degree of glucose intolerance first recognized during pregnancy. Insulin may or may not be needed. People do not go back and forth between types 1 and 2 diabetes. PTS: 1 DIF: Cognitive Level: Knowledge REF: 268 OBJ: Nursing Process: Assessment MSC: Client Needs: Physiologic Integrity

Autoimmune disorders often occur during pregnancy because a large percentage of women with an autoimmune disorder are of childbearing age. Identify all disorders that fall into the category of collagen vascular disease. a. Multiple sclerosis b. Systemic lupus erythematosus c. Antiphospholipid syndrome d. Rheumatoid arthritis e. Myasthenia gravis

ANS: B, C, D, E Multiple sclerosis is not an autoimmune disorder. This patchy demyelinization of the spinal cord may be a viral disorder. Autoimmune disorders (collagen vascular disease) make up a large group of conditions that disrupt the function of the immune system of the body. They include those listed, as well as systemic sclerosis. PTS: 1 DIF: Cognitive Level: Comprehension REF: 296 OBJ: Nursing Process: Assessment MSC: Client Needs: Health Promotion and Maintenance COMPLETION

22. A woman presents to the emergency department with complaints of bleeding and cramping. The initial nursing history is significant for a last menstrual period 6 weeks ago. On sterile speculum examination, the primary care provider finds that the cervix is closed. The anticipated plan of care for this woman would be based on a probable diagnosis of which type of spontaneous abortion? a. Incomplete c. Threatened b. Inevitable d. Septic

ANS: C A woman with a threatened abortion presents with spotting, mild cramps, and no cervical dilation. A woman with an incomplete abortion would present with heavy bleeding, mild to severe cramping, and cervical dilation. An inevitable abortion manifests with the same symptoms as an incomplete abortion: heavy bleeding, mild to severe cramping, and cervical dilation. A woman with a septic abortion presents with malodorous bleeding and typically a dilated cervix.

36. What condition indicates concealed hemorrhage when the patient experiences an abruptio placentae? a. Decrease in abdominal pain c. Hard, boardlike abdomen b. Bradycardia d. Decrease in fundal height

ANS: C Concealed hemorrhage occurs when the edges of the placenta do not separate. The formation of a hematoma behind the placenta and subsequent infiltration of the blood into the uterine muscle results in a very firm, boardlike abdomen. Abdominal pain may increase. The patient will have shock symptoms that include tachycardia. As bleeding occurs, the fundal height will increase.

28. A woman arrives for evaluation of her symptoms, which include a missed period, adnexal fullness, tenderness, and dark red vaginal bleeding. On examination the nurse notices an ecchymotic blueness around the woman's umbilicus and recognizes this assessment finding as: a. Normal integumentary changes associated with pregnancy. b. Turner's sign associated with appendicitis. c. Cullen's sign associated with a ruptured ectopic pregnancy. d. Chadwick's sign associated with early pregnancy.

ANS: C Cullen's sign, the blue ecchymosis seen in the umbilical area, indicates hematoperitoneum associated with an undiagnosed ruptured intraabdominal ectopic pregnancy. Linea nigra on the abdomen is the normal integumentary change associated with pregnancy. It manifests as a brown, pigmented, vertical line on the lower abdomen. Turner's sign is ecchymosis in the flank area, often associated with pancreatitis. Chadwick's sign is the blue-purple color of the cervix that may be seen during or around the eighth week of pregnancy.

7. The labor of a pregnant woman with preeclampsia is going to be induced. Before initiating the Pitocin infusion, the nurse reviews the woman's latest laboratory test findings, which reveal a platelet count of 90,000, an elevated aspartate transaminase (AST) level, and a falling hematocrit. The nurse notifies the physician because the laboratory results are indicative of: a. Eclampsia. b. Disseminated intravascular coagulation (DIC). c. HELLP syndrome. d. Idiopathic thrombocytopenia.

ANS: C HELLP syndrome is a laboratory diagnosis for a variant of severe preeclampsia that involves hepatic dysfunction characterized by hemolysis (H), elevated liver enzymes (EL), and low platelets (LP). Eclampsia is determined by the presence of seizures. DIC is a potential complication associated with HELLP syndrome. Idiopathic thrombocytopenia is the presence of low platelets of unknown cause and is not associated with preeclampsia.

31. In providing nutritional counseling for the pregnant woman experiencing cholecystitis, the nurse would: a. Assess the woman's dietary history for adequate calories and proteins. b. Instruct the woman that the bulk of calories should come from proteins. c. Instruct the woman to eat a low-fat diet and avoid fried foods. d. Instruct the woman to eat a low-cholesterol, low-salt diet.

ANS: C Instructing the woman to eat a low-fat diet and avoid fried foods is appropriate nutritional counseling for this client. Caloric and protein intake do not predispose a woman to the development of cholecystitis. The woman should be instructed to limit protein intake and choose foods that are high in carbohydrates. A low-cholesterol diet may be the result of limiting fats. However, a low-salt diet is not indicated.

20. Magnesium sulfate is given to women with preeclampsia and eclampsia to: a. Improve patellar reflexes and increase respiratory efficiency. b. Shorten the duration of labor. c. Prevent and treat convulsions. d. Prevent a boggy uterus and lessen lochial flow.

ANS: C Magnesium sulfate is the drug of choice to prevent convulsions, although it can generate other problems. Loss of patellar reflexes and respiratory depression are signs of magnesium toxicity. Magnesium sulfate can increase the duration of labor. Women are at risk for a boggy uterus and heavy lochial flow as a result of magnesium sulfate therapy.

25. Methotrexate is recommended as part of the treatment plan for which obstetric complication? a. Complete hydatidiform mole c. Unruptured ectopic pregnancy b. Missed abortion d. Abruptio placentae

ANS: C Methotrexate is an effective, nonsurgical treatment option for a hemodynamically stable woman whose ectopic pregnancy is unruptured and less than 4 cm in diameter. Methotrexate is not indicated or recommended as a treatment option for complete hydatidiform mole, missed abortion, and abruptio placentae.

6. A primigravida is being monitored in her prenatal clinic for preeclampsia. What finding should concern her nurse? a. Blood pressure (BP) increase to 138/86 mm Hg b. Weight gain of 0.5 kg during the past 2 weeks c. A dipstick value of 3+ for protein in her urine d. Pitting pedal edema at the end of the day

ANS: C Proteinuria is defined as a concentration of 1+ or greater via dipstick measurement. A dipstick value of 3+ should alert the nurse that additional testing or assessment should be made. Generally, hypertension is defined as a BP of 140/90 or an increase in systolic pressure of 30 mm Hg or in diastolic pressure of 15 mm Hg. Preeclampsia may be manifested as a rapid weight gain of more than 2 kg in 1 week. Edema occurs in many normal pregnancies and in women with preeclampsia. Therefore, the presence of edema is no longer considered diagnostic of preeclampsia.

9. A pregnant woman has been receiving a magnesium sulfate infusion for treatment of severe preeclampsia for 24 hours. On assessment the nurse finds the following vital signs: temperature of 37.3° C, pulse rate of 88 beats/min, respiratory rate of 10 breaths/min, blood pressure (BP) of 148/90 mm Hg, absent deep tendon reflexes, and no ankle clonus. The client complains, "I'm so thirsty and warm." The nurse: a. Calls for a stat magnesium sulfate level. b. Administers oxygen. c. Discontinues the magnesium sulfate infusion. d. Prepares to administer hydralazine.

ANS: C The client is displaying clinical signs and symptoms of magnesium toxicity. Magnesium should be discontinued immediately. In addition, calcium gluconate, the antidote for magnesium, may be administered. Hydralazine is an antihypertensive commonly used to treat hypertension in severe preeclampsia. Typically it is administered for a systolic BP greater than 160 mm Hg or a diastolic BP greater than 110 mm Hg.

40. What finding on a prenatal visit at 10 weeks could suggest a hydatidiform mole? a. Complaint of frequent mild nausea b. Blood pressure of 120/80 mm Hg c. Fundal height measurement of 18 cm d. History of bright red spotting for 1 day, weeks ago

ANS: C The uterus in a hydatidiform molar pregnancy is often larger than would be expected on the basis of the duration of the pregnancy. Nausea increases in a molar pregnancy because of the increased production of hCG. A woman with a molar pregnancy may have early-onset pregnancy-induced hypertension. In the patient's history, bleeding is normally described as brownish.

21. Preeclampsia is a unique disease process related only to human pregnancy. The exact cause of this condition continues to elude researchers. The American College of Obstetricians and Gynecologists has developed a comprehensive list of risk factors associated with the development of preeclampsia. Which client exhibits the greatest number of these risk factors? a. A 30-year-old obese Caucasian with her third pregnancy b. A 41-year-old Caucasian primigravida c. An African-American client who is 19 years old and pregnant with twins d. A 25-year-old Asian-American whose pregnancy is the result of donor insemination

ANS: C Three risk factors are present for this woman. She is of African-American ethnicity, is at the young end of the age distribution, and has a multiple pregnancy. In planning care for this client the nurse must monitor blood pressure frequently and teach the woman regarding early warning signs. The 30-year-old client only has one known risk factor, obesity. Age distribution appears to be U-shaped, with women less than 20 years and more than 40 years being at greatest risk. Preeclampsia continues to be seen more frequently in primigravidas; this client is a multigravida woman. Two risk factors are present for the 41-year-old client. Her age and status as a primigravida put her at increased risk for preeclampsia. Caucasian women are at a lower risk than African-American women. The Asian-American client exhibits only one risk factor. Pregnancies that result from donor insemination, oocyte donation, and embryo donation are at an increased risk of developing preeclampsia.

The nurse providing care for a woman with gestational diabetes understands that a laboratory test for glycosylated hemoglobin Alc: a. Is now done for all pregnant women, not just those with or likely to have diabetes. b. Is a snapshot of glucose control at the moment. c. Would be considered evidence of good diabetes control with a result of 5% to 6%. d. Is done on the patients urine, not her blood.

ANS: C A score of 5% to 6% indicates good control. This is an extra test for diabetic women, not one done for all pregnant women. This test defines glycemic control over the previous 4 to 6 weeks. Glycosylated hemoglobin level tests are done on the blood. PTS: 1 DIF: Cognitive Level: Comprehension REF: 273 OBJ: Nursing Process: Evaluation MSC: Client Needs: Health Promotion and Maintenance

23. Some pregnant clients may complain of changes in their voice and impaired hearing. The nurse can tell these clients that these are common reactions to: a. A decreased estrogen level. b. Displacement of the diaphragm, resulting in thoracic breathing. c. Congestion and swelling, which occur because the upper respiratory tract has become more vascular. d. Increased blood volume.

ANS: C Estrogen levels increase, causing the upper respiratory tract to become more vascular producing swelling and congestion in the nose and ears leading to voice changes and impaired hearing. The diaphragm is displaced, and the volume of blood is increased. However, the main concern is increased estrogen levels.

To manage her diabetes appropriately and ensure a good fetal outcome, the pregnant woman with diabetes will need to alter her diet by: a. Eating six small equal meals per day. b. Reducing carbohydrates in her diet. c. Eating her meals and snacks on a fixed schedule. d. Increasing her consumption of protein.

ANS: C Having a fixed meal schedule will provide the woman and the fetus with a steadier blood sugar level, provide better balance with insulin administration, and help prevent complications. It is more important to have a fixed meal schedule than equal division of food intake. Approximately 45% of the food eaten should be in the form of carbohydrates. PTS: 1 DIF: Cognitive Level: Comprehension REF: 274 OBJ: Nursing Process: Planning MSC: Client Needs: Health Promotion and Maintenance

Nursing intervention for the pregnant diabetic patient is based on the knowledge that the need for insulin: a. Increases throughout pregnancy and the postpartum period. b. Decreases throughout pregnancy and the postpartum period. c. Varies depending on the stage of gestation. d. Should not change because the fetus produces its own insulin.

ANS: C Insulin needs decrease during the first trimester, when nausea, vomiting, and anorexia are a factor. They increase during the second and third trimesters, when the hormones of pregnancy create insulin resistance in maternal cells. Insulin needs increase during the second and third trimesters, when the hormones of pregnancy create insulin resistance in maternal cells. The insulin needs change throughout the different stages of pregnancy. PTS: 1 DIF: Cognitive Level: Comprehension REF: 269 OBJ: Nursing Process: Implementation MSC: Client Needs: Physiologic Integrity

20. To reassure and educate pregnant clients about changes in their breasts, nurses should be aware that: a. The visibility of blood vessels that form an intertwining blue network indicates full function of Montgomery's tubercles and possibly infection of the tubercles. b. The mammary glands do not develop until 2 weeks before labor. c. Lactation is inhibited until the estrogen level declines after birth. d. Colostrum is the yellowish oily substance used to lubricate the nipples for breastfeeding

ANS: C Lactation is inhibited until after birth. The visible blue network of blood vessels is a normal outgrowth of a richer blood supply. The mammary glands are functionally complete by midpregnancy. Colostrum is a creamy, white-to-yellow premilk fluid that can be expressed from the nipples before birth.

A patient at 24 weeks of gestation contacts the nurse at her obstetric provider's office to complain that she has cravings for dirt and gravel. The nurse is aware that this condition is known as ________ and may indicate anemia. a. Ptyalism b. Pyrosis c. Pica d. Decreased peristalsis

ANS: C Pica (a desire to eat nonfood substances) is an indication of iron deficiency and should be evaluated. Ptyalism (excessive salivation), pyrosis (heartburn), and decreased peristalsis are normal findings of gastrointestinal change during pregnancy. Food cravings during pregnancy are normal.

Which heart condition is not a contraindication for pregnancy? a. Peripartum cardiomyopathy b. Eisenmenger syndrome c. Heart transplant d. All of these contraindicate pregnancy.

ANS: C Pregnancy is contraindicated for peripartum cardiomyopathy and Eisenmenger syndrome. Women who have had heart transplants are successfully having babies. However, conception should be postponed for at least 1 year after transplantation. PTS: 1 DIF: Cognitive Level: Comprehension REF: 287 OBJ: Nursing Process: Assessment MSC: Client Needs: Health Promotion and Maintenance

Metabolic changes throughout pregnancy that affect glucose and insulin in the mother and the fetus are complicated but important to understand. Nurses should understand that: a. Insulin crosses the placenta to the fetus only in the first trimester, after which the fetus secretes its own. b. Women with insulin-dependent diabetes are prone to hyperglycemia during the first trimester because they are consuming more sugar. c. During the second and third trimesters, pregnancy exerts a diabetogenic effect that ensures an abundant supply of glucose for the fetus. d. Maternal insulin requirements steadily decline during pregnancy.

ANS: C Pregnant women develop increased insulin resistance during the second and third trimesters. Insulin never crosses the placenta; the fetus starts making its own insulin around the tenth week. As a result of normal metabolic changes during pregnancy, insulin-dependent women are prone to hypoglycemia (low levels). Maternal insulin requirements may double or quadruple by the end of pregnancy. PTS: 1 DIF: Cognitive Level: Comprehension REF: 270 OBJ: Nursing Process: Assessment MSC: Client Needs: Physiologic Integrity

Which factor is known to increase the risk of gestational diabetes mellitus? a. Underweight before pregnancy b. Maternal age younger than 25 years c. Previous birth of large infant d. Previous diagnosis of type 2 diabetes mellitus

ANS: C Previous birth of a large infant suggests gestational diabetes mellitus. Obesity (BMI of 30 or greater) creates a higher risk for gestational diabetes. A woman younger than 25 years generally is not at risk for gestational diabetes mellitus. The person with type 2 diabetes mellitus already has diabetes and will continue to have it after pregnancy. Insulin may be required during pregnancy because oral hypoglycemia drugs are contraindicated during pregnancy. PTS: 1 DIF: Cognitive Level: Comprehension REF: 279 OBJ: Nursing Process: Assessment MSC: Client Needs: Health Promotion and Maintenance

A woman with asthma is experiencing a postpartum hemorrhage. Which drug would not be used to treat her bleeding because it may exacerbate her asthma? a. Pitocin b. Nonsteroidal antiinflammatory drugs (NSAIDs) c. Hemabate d. Fentanyl

ANS: C Prostaglandin derivatives should not be used to treat women with asthma, because they may exacerbate symptoms. Pitocin would be the drug of choice to treat this womans bleeding because it would not exacerbate her asthma. NSAIDs are not used to treat bleeding. Fentanyl is used to treat pain, not bleeding. PTS: 1 DIF: Cognitive Level: Analysis REF: 292 OBJ: Nursing Process: Planning MSC: Client Needs: Physiologic Integrity

_____ use/abuse during pregnancy causes vasoconstriction and decreased placental perfusion, resulting in maternal and neonatal complications. a. Alcohol b. Caffeine c. Tobacco d. Chocolate

ANS: C Smoking in pregnancy is known to cause a decrease in placental perfusion and has serious health risks, including bleeding complications, low birth weight, prematurity, miscarriage, stillbirth, and sudden infant death syndrome. Prenatal alcohol exposure is the single greatest preventable cause of mental retardation. Alcohol use during pregnancy can cause high blood pressure, miscarriage, premature birth, stillbirth, and anemia. Caffeine and chocolate may safely be consumed in small quantities during pregnancy. PTS: 1 DIF: Cognitive Level: Knowledge REF: 298 OBJ: Nursing Process: Assessment MSC: Client Needs: Health Promotion and Maintenance

The most common neurologic disorder accompanying pregnancy is: a. Eclampsia. b. Bells palsy. c. Epilepsy. d. Multiple sclerosis.

ANS: C The effects of pregnancy on epilepsy are unpredictable. Eclampsia sometimes may be confused with epilepsy, which is the most common neurologic disorder accompanying pregnancy. Bells palsy is a form of facial paralysis. Multiple sclerosis is a patchy demyelinization of the spinal cord that does not affect the normal course of pregnancy or birth. PTS: 1 DIF: Cognitive Level: Knowledge REF: 294 OBJ: Nursing Process: Planning MSC: Client Needs: Physiologic Integrity

In caring for a pregnant woman with sickle cell anemia, the nurse is aware that signs and symptoms of sickle cell crisis include: a. Anemia. b. Endometritis. c. Fever and pain. d. Urinary tract infection.

ANS: C Women with sickle cell anemia have recurrent attacks (crisis) of fever and pain, most often in the abdomen, joints, and extremities. These attacks are attributed to vascular occlusion when RBCs assume the characteristic sickled shape. Crises are usually triggered by dehydration, hypoxia, or acidosis. Women with sickle cell anemia are not iron deficient. Therefore, routine iron supplementation, even that found in prenatal vitamins, should be avoided in order to prevent iron overload. Women with sickle cell trait usually are at greater risk for postpartum endometritis (uterine wall infection); however, this is not likely to occur in pregnancy and is not a sign of crisis. These women are at an increased risk for UTIs; however, this is not an indication of sickle cell crisis. PTS: 1 DIF: Cognitive Level: Comprehension REF: 291 OBJ: Nursing Process: Assessment MSC: Client Needs: Physiologic Integrity

33. During pregnancy, many changes occur as a direct result of the presence of the fetus. Which of these adaptations meet this criteria? [select all that apply] a. Leukorrhea b. Development of the operculum c. Quickening d. Ballottement e. Lightening

ANS: C, D, E Leukorrhea is a white or slightly gray vaginal discharge that develops in response to cervical stimulation by estrogen and progesterone. Quickening is the first recognition of fetal movements or "feeling life." Quickening is often described as a flutter and is felt earlier in multiparous women than in primiparas. Lightening occurs when the fetus begins to descend into the pelvis. This occurs 2 weeks before labor in the nullipara and at the start of labor in the multipara. Mucus fills the cervical canal creating a plug otherwise known as the operculum. The operculum acts as a barrier against bacterial invasion during the pregnancy. Passive movement of the unengaged fetus is referred to as ballottement.

35. A placenta previa in which the placental edge just reaches the internal os is more commonly known as: a. Total c. Complete b. Partial d. Marginal

ANS: D A placenta previa that does not cover any part of the cervix is termed marginal. With a total placenta previa, the placenta completely covers the os. When the patient experiences a partial placenta previa, the lower border of the placenta is within 3 cm of the internal cervical os but does not completely cover the os. A complete placenta previa is termed total. The placenta completely covers the internal cervical os.

13. Your patient has been receiving magnesium sulfate for 20 hours for treatment of preeclampsia. She just delivered a viable infant girl 30 minutes ago. What uterine findings would you expect to observe/assess in this client? a. Absence of uterine bleeding in the postpartum period b. A fundus firm below the level of the umbilicus c. Scant lochia flow d. A boggy uterus with heavy lochia flow

ANS: D Because of the tocolytic effects of magnesium sulfate, this patient most likely would have a boggy uterus with increased amounts of bleeding and a heavy lochia flow in the postpartum period.

29. As related to the care of the patient with miscarriage, nurses should be aware that: a. It is a natural pregnancy loss before labor begins. b. It occurs in fewer than 5% of all clinically recognized pregnancies. c. It often can be attributed to careless maternal behavior such as poor nutrition or excessive exercise. d. If it occurs before the twelfth week of pregnancy, it may manifest only as moderate discomfort and blood loss.

ANS: D Before the sixth week the only evidence may be a heavy menstrual flow. After the twelfth week more severe pain, similar to that of labor, is likely. Miscarriage is a natural pregnancy loss, but by definition it occurs before 20 weeks of gestation, before the fetus is viable. Miscarriages occur in approximately 10% to 15% of all clinically recognized pregnancies. Miscarriage can be caused by a number of disorders or illnesses outside of the mother's control or knowledge.

18. Nurses should be aware that chronic hypertension: a. Is defined as hypertension that begins during pregnancy and lasts for the duration of pregnancy. b. Is considered severe when the systolic blood pressure (BP) is greater than 140 mm Hg or the diastolic BP is greater than 90 mm Hg. c. Is general hypertension plus proteinuria. d. Can occur independently of or simultaneously with gestational hypertension.

ANS: D Hypertension is present before pregnancy or diagnosed before 20 weeks of gestation and persists longer than 6 weeks postpartum. The range for hypertension is systolic BP greater than 140 mm Hg or diastolic BP greater than 90 mm Hg. It becomes severe with a diastolic BP of 110 mm Hg or higher. Proteinuria is an excessive concentration of protein in the urine. It is a complication of hypertension, not a defining characteristic.

8. A woman with preeclampsia has a seizure. The nurse's primary duty during the seizure is to: a. Insert an oral airway. b. Suction the mouth to prevent aspiration. c. Administer oxygen by mask. d. Stay with the client and call for help.

ANS: D If a client becomes eclamptic, the nurse should stay her and call for help. Insertion of an oral airway during seizure activity is no longer the standard of care. The nurse should attempt to keep the airway patent by turning the client's head to the side to prevent aspiration. Once the seizure has ended, it may be necessary to suction the client's mouth. Oxygen would be administered after the convulsion has ended.

41. A 32-year-old primigravida is admitted with a diagnosis of ectopic pregnancy. Nursing care is based on the knowledge that: a. Bed rest and analgesics are the recommended treatment. b. She will be unable to conceive in the future. c. A D&C will be performed to remove the products of conception. d. Hemorrhage is the major concern.

ANS: D Severe bleeding occurs if the fallopian tube ruptures. The recommended treatment is to remove the pregnancy before rupture in order to prevent hemorrhaging. If the tube must be removed, the woman's fertility will decrease; however, she will not be infertile.

11. A woman at 39 weeks of gestation with a history of preeclampsia is admitted to the labor and birth unit. She suddenly experiences increased contraction frequency of every 1 to 2 minutes; dark red vaginal bleeding; and a tense, painful abdomen. The nurse suspects the onset of: a. Eclamptic seizure. c. Placenta previa. b. Rupture of the uterus. d. Placental abruption.

ANS: D Uterine tenderness in the presence of increasing tone may be the earliest finding of premature separation of the placenta (abruptio placentae or placental abruption). Women with hypertension are at increased risk for an abruption. Eclamptic seizures are evidenced by the presence of generalized tonic-clonic convulsions. Uterine rupture manifests as hypotonic uterine activity, signs of hypovolemia, and in many cases the absence of pain. Placenta previa manifests with bright red, painless vaginal bleeding.

A woman with gestational diabetes has had little or no experience reading and interpreting glucose levels. She shows the nurse her readings for the past few days. Which one should the nurse tell her indicates a need for adjustment (insulin or sugar)? a. 75 mg/dL before lunch. This is low; better eat now. b. 115 mg/dL 1 hour after lunch. This is a little high; maybe eat a little less next time. c. 115 mg/dL 2 hours after lunch; This is too high; it is time for insulin. d. 60 mg/dL just after waking up from a nap. This is too low; maybe eat a snack before going to sleep.

ANS: D 60 mg/dL after waking from a nap is too low. During hours of sleep glucose levels should not be less than 70 mg/dL. Snacks before sleeping can be helpful. The premeal acceptable range is 65 to 95 mg/dL. The readings 1 hour after a meal should be less than 140 mg/dL. Two hours after eating, the readings should be less than 120 mg/dL. PTS: 1 DIF: Cognitive Level: Application REF: 277 OBJ: Nursing Process: Evaluation MSC: Client Needs: Health Promotion and Maintenance

When caring for a pregnant woman with cardiac problems, the nurse must be alert for signs and symptoms of cardiac decompensation, which include: a. A regular heart rate and hypertension. b. An increased urinary output, tachycardia, and dry cough. c. Shortness of breath, bradycardia, and hypertension. d. Dyspnea; crackles; and an irregular, weak pulse.

ANS: D Signs of cardiac decompensation include dyspnea; crackles; an irregular, weak, rapid pulse; rapid respirations; a moist, frequent cough; generalized edema; increasing fatigue; and cyanosis of the lips and nail beds. A regular heart rate and hypertension are not generally associated with cardiac decompensation. Tachycardia would indicate cardiac decompensation, but increased urinary output and a dry cough would not. Shortness of breath would indicate cardiac decompensation, but bradycardia and hypertension would not. PTS: 1 DIF: Cognitive Level: Comprehension REF: 288 OBJ: Nursing Process: Assessment MSC: Client Needs: Physiologic Integrity

22. To reassure and educate their pregnant clients about changes in their blood pressure, maternity nurses should be aware that: a. A blood pressure cuff that is too small produces a reading that is too low; a cuff that is too large produces a reading that is too high. b. Shifting the client's position and changing from arm to arm for different measurements produces the most accurate composite blood pressure reading at each visit. c. The systolic blood pressure increases slightly as pregnancy advances; the diastolic pressure remains constant. d. Compression of the iliac veins and inferior vena cava by the uterus contributes to hemorrhoids in the later stage of term pregnancy.

ANS: D Compression of the iliac veins and inferior vena cava also leads to varicose veins in the legs and vulva. The tightness of a cuff that is too small produces a reading that is too high; similarly the looseness of a cuff that is too large results in a reading that is too low. Because maternal positioning affects readings, blood pressure measurements should be obtained in the same arm and with the woman in the same position. The systolic blood pressure generally remains constant but may decline slightly as pregnancy advances. The diastolic blood pressure first decreases and then gradually increases.

Since the gene for cystic fibrosis was identified in 1989, data can be collected for the purposes of genetic counseling for couples regarding carrier status. According to statistics, how often does cystic fibrosis occur in Caucasian live births? a. 1 in 100 b. 1 in 1200 c. 1 in 2500 d. 1 in 3000

ANS: D Cystic fibrosis occurs in about 1 in 3000 Caucasian live births. PTS: 1 DIF: Cognitive Level: Comprehension REF: 293 OBJ: Nursing Process: Assessment MSC: Client Needs: Health Promotion and Maintenance

16. Human chorionic gonadotropin (hCG) is an important biochemical marker for pregnancy and the basis for many tests. A maternity nurse should be aware that: a. hCG can be detected 2.5 weeks after conception. b. The hCG level increases gradually and uniformly throughout pregnancy. c. Much lower than normal increases in the level of hCG may indicate a postdate pregnancy. d. A higher than normal level of hCG may indicate an ectopic pregnancy or Down syndrome.

ANS: D Higher levels also could be a sign of multiple gestation. hCG can be detected 7 to 8 days after conception. The hCG level fluctuates during pregnancy: peaking, declining, stabilizing, and increasing again. Abnormally slow increases may indicate impending miscarriage.

18. To reassure and educate pregnant clients about changes in the cervix, vagina, and position of the fetus, nurses should be aware that: a. Because of a number of changes in the cervix, abnormal Papanicolaou (Pap) tests are much easier to evaluate. b. Quickening is a technique of palpating the fetus to engage it in passive movement. c. The deepening color of the vaginal mucosa and cervix (Chadwick's sign) usually appears in the second trimester or later as the vagina prepares to stretch during labor. d. Increased vascularity of the vagina increases sensitivity and may lead to a high degree of arousal, especially in the second trimester.

ANS: D Increased sensitivity and an increased interest in sex sometimes go together. This frequently occurs during the second trimester. Cervical changes make evaluation of abnormal Pap tests more difficult. Quickening is the first recognition of fetal movements by the mother. Ballottement is a technique used to palpate the fetus. Chadwick's sign appears from the sixth to eighth weeks.

What form of heart disease in women of childbearing years usually has a benign effect on pregnancy? a. Cardiomyopathy b. Rheumatic heart disease c. Congenital heart disease d. Mitral valve prolapse

ANS: D Mitral valve prolapse is a benign condition that is usually asymptomatic. Cardiomyopathy produces congestive heart failure during pregnancy. Rheumatic heart disease can lead to heart failure during pregnancy. Some congenital heart diseases produce pulmonary hypertension or endocarditis during pregnancy. PTS: 1 DIF: Cognitive Level: Knowledge REF: 284 OBJ: Nursing Process: Assessment MSC: Client Needs: Physiologic Integrity

Which statement about a condition of pregnancy is accurate? a. Insufficient salivation (ptyalism) is caused by increases in estrogen. b. Acid indigestion (pyrosis) begins early but declines throughout pregnancy. c. Hyperthyroidism often develops (temporarily) because hormone production increases. d. Nausea and vomiting rarely have harmful effects on the fetus and may be beneficial.

ANS: D Normal nausea and vomiting rarely produce harmful effects, and nausea and vomiting periods may be less likely to result in miscarriage or preterm labor. Ptyalism is excessive salivation, which may be caused by a decrease in unconscious swallowing or stimulation of the salivary glands. Pyrosis begins in the first trimester and intensifies through the third trimester. Increased hormone production does not lead to hyperthyroidism in pregnant women.

Glucose metabolism is profoundly affected during pregnancy because: a. Pancreatic function in the islets of Langerhans is affected by pregnancy. b. The pregnant woman uses glucose at a more rapid rate than the nonpregnant woman. c. The pregnant woman increases her dietary intake significantly. d. Placental hormones are antagonistic to insulin, thus resulting in insulin resistance.

ANS: D Placental hormones, estrogen, progesterone, and human placental lactogen (HPL) create insulin resistance. Insulin also is broken down more quickly by the enzyme placental insulinase. Pancreatic functioning is not affected by pregnancy. The glucose requirements differ because of the growing fetus. The pregnant woman should increase her intake by 200 calories a day. PTS: 1 DIF: Cognitive Level: Comprehension REF: 279 OBJ: Nursing Process: Assessment MSC: Client Needs: Physiologic Integrity

11. The nurse caring for the pregnant client must understand that the hormone essential for maintaining pregnancy is: a. Estrogen. b. Human chorionic gonadotropin (hCG). c. Oxytocin. d. Progesterone.

ANS: D Progesterone is essential for maintaining pregnancy; it does so by relaxing smooth muscles. This reduces uterine activity and prevents miscarriage. Estrogen plays a vital role in pregnancy, but it is not the primary hormone for maintaining pregnancy. hCG levels increase at implantation but decline after 60 to 70 days. Oxytocin stimulates uterine contractions.

7. Numerous changes in the integumentary system occur during pregnancy. Which change persists after birth? a. Epulis b. Chloasma c. Telangiectasia d. Striae gravidarum

ANS: D Striae gravidarum, or stretch marks, reflect separation within the underlying connective tissue of the skin. They usually fade after birth, although they never disappear completely. An epulis is a red, raised nodule on the gums that bleeds easily. Chloasma, or mask of pregnancy, is a blotchy, brown hyperpigmentation of the skin over the cheeks, nose, and forehead, especially in dark-complexioned pregnant women. Chloasma usually fades after the birth. Telangiectasia, or vascular spiders, are tiny, star-shaped or branchlike, slightly raised, pulsating end-arterioles usually found on the neck, thorax, face, and arms. They occur as a result of elevated levels of circulating estrogen. These usually disappear after birth.

During a physical assessment of an at-risk client, the nurse notes generalized edema, crackles at the base of the lungs, and some pulse irregularity. These are most likely signs of: a. Euglycemia. b. Rheumatic fever. c. Pneumonia. d. Cardiac decompensation.

ANS: D Symptoms of cardiac decompensation may appear abruptly or gradually. Euglycemia is a condition of normal glucose levels. These symptoms indicate cardiac decompensation. Rheumatic fever can cause heart problems, but it does not manifest with these symptoms, which indicate cardiac decompensation. Pneumonia is an inflammation of the lungs and would not likely generate these symptoms, which indicate cardiac decompensation. PTS: 1 DIF: Cognitive Level: Analysis REF: 288 OBJ: Nursing Process: Assessment MSC: Client Needs: Physiologic Integrity

In planning for the care of a 30-year-old woman with pregestational diabetes, the nurse recognizes that the most important factor affecting pregnancy outcome is the: a. Mothers age. b. Number of years since diabetes was diagnosed. c. Amount of insulin required prenatally. d. Degree of glycemic control during pregnancy.

ANS: D Women with excellent glucose control and no blood vessel disease should have good pregnancy outcomes. PTS: 1 DIF: Cognitive Level: Comprehension REF: 273 OBJ: Nursing Process: Planning MSC: Client Needs: Health Promotion and Maintenance

Congenital disorders refer to conditions that are present at birth. These disorders may be inherited and caused by environmental factors or maternal malnutrition. Toxic exposures have the greatest effect on development between 15 and 60 days of gestation. For the nurse to be able to conduct a complete assessment of the newly pregnant client, she should understand the significance of exposure to known human teratogens. These include(Select all that apply): a. Infections. b. Radiation. c. Maternal conditions. d. Drugs. e. Chemicals.

All

A woman who is 16 weeks pregnant asks the nurse, "Is it possible to tell by ultrasound if the baby is a boy or girl yet?" The best answer is: a. "A baby's sex is determined as soon as conception occurs." b. "The baby has developed enough that we can determine the sex by examining the genitals through ultrasound." c. "Boys and girls look alike until about 20 weeks after conception, and then they begin to look different." d. "It might be possible to determine your baby's sex, but the external organs look very similar right now."

B

As relates to the structure and function of the placenta, the maternity nurse should be aware that: a. As the placenta widens, it gradually thins to allow easier passage of air and nutrients. b. As one of its early functions, the placenta acts as an endocrine gland. c. The placenta is able to keep out most potentially toxic substances such as cigarette smoke to which the mother is exposed. d. Optimal blood circulation is achieved through the placenta when the woman is lying on her back or standing.

B

At approximately _____ weeks of gestation, lecithin is forming on the alveolar surfaces, the eyelids open, and the fetus measures approximately 27 cm crown to rump and weighs approximately 1110 g. a. 20 b. 28 c. 24 d. 30

B

In presenting to obstetric nurses interested in genetics, the genetic nurse identifies the primary risk(s) associated with genetic testing as: a. Anxiety and altered family relationships. b. Denial of insurance benefits. c. High false-positive results associated with genetic testing. d. Ethnic and socioeconomic disparity associated with genetic testing.

B

The measurement of lecithin in relation to sphingomyelin (L/S ratio) is used to determine fetal lung maturity. Which ratio reflects maturity of the lungs? a. 1.4:1 b. 2:1 c. 1.8:1 d. 1:1

B

With regard to chromosome abnormalities, nurses should be aware that: a. They occur in approximately 10% of newborns. b. Abnormalities of number are the leading cause of pregnancy loss. c. Down syndrome is a result of an abnormal chromosome structure. d. Unbalanced translocation results in a mild abnormality that the child will outgrow.

B

You are a maternal-newborn nurse caring for a mother who just delivered a baby born with Down syndrome. What nursing diagnosis would be the most essential in caring for the mother of this infant? a. Disturbed body image c. Anxiety b. Interrupted family processes d. Risk for injury

B

What type of cultural concern is the most likely deterrent to many women seeking prenatal care? A. Religion B. Modesty C. Ignorance D. Belief that physicians are evil

B (A concern for modesty is a deterrent to many women seeking prenatal care. For some women, exposing body parts, especially to a man, is considered a major violation of their modesty. Many cultural variations are found in prenatal care. Even if the prenatal care described is familiar to a woman, some practices may conflict with the beliefs and practices of a subculture group to which she belongs.)

A woman who is 32 weeks' pregnant is informed by the nurse that a danger sign of pregnancy could be: A. Constipation. B. Alteration in the pattern of fetal movement. C. Heart palpitations. D. Edema in the ankles and feet at the end of the day.

B (An alteration in the pattern or amount of fetal movement may indicate fetal jeopardy. Constipation, heart palpitations, and ankle and foot edema are normal discomforts of pregnancy that occur in the second and third trimesters.)

With regard to the initial physical examination of a woman beginning prenatal care, maternity nurses should be cognizant of: A. Only women who show physical signs or meet the sociologic profile should be assessed for physical abuse. B. The woman should empty her bladder before the pelvic examination is performed. C. The distribution, amount, and quality of body hair are of no particular importance. D. The size of the uterus is discounted in the initial examination.

B (An empty bladder facilitates the examination; this is also an opportunity to get a urine sample easily for a number of tests. All women should be assessed for a history of physical abuse, particularly because the likelihood of abuse increases during pregnancy. Noting body hair is important because body hair reflects nutritional status, endocrine function, and hygiene. Particular attention is paid to the size of the uterus because it is an indication of the duration of gestation.)

A 3-year-old girl's mother is 6 months pregnant. What concern is this child likely to verbalize? A. How the baby will "get out" B. What the baby will eat C. Whether her mother will die D. What color eyes the baby has

B (By age 3 or 4, children like to be told the story of their own beginning and accept its comparison with the present pregnancy. They like to listen to the fetal heartbeat and feel the baby move. Sometimes they worry about how the baby is being fed and what it wears. School-age children take a more clinical interest in their mother's pregnancy and may want to know, "How did the baby get in there?" and "How will it get out?" Whether her mother will die does not tend to be the focus of a child's questions about the impending birth of a sibling. The baby's eye color does not tend to be the focus of children's questions about the impending birth of a sibling.)

A woman who is 32 weeks pregnant is informed by the nurse that a danger sign of pregnancy could be: A. Constipation. B. Alteration in the pattern of fetal movement. C. Heart palpitations. D. Edema in the ankles and feet at the end of the day.

B (Constipation is a normal discomfort of pregnancy that occurs in the second and third trimesters. An alteration in the pattern or amount of fetal movement may indicate fetal jeopardy. Heart palpitations are a normal change related to pregnancy. This is most likely to occur during the second and third trimesters. As the pregnancy progresses, edema in the ankles and feet at the end of the day is not uncommon.)

During the first trimester, a woman can expect which of the following changes in her sexual desire? A. An increase, because of enlarging breasts B. A decrease, because of nausea and fatigue C. No change D. An increase, because of increased levels of female hormones

B (Maternal physiologic changes such as breast enlargement, nausea, fatigue, abdominal changes, perineal enlargement, leukorrhea, pelvic vasocongestion, and orgasmic responses may affect sexuality and sexual expression. Libido may be depressed in the first trimester but often increases during the second and third trimesters. During pregnancy, the breasts may become enlarged and tender; this tends to interfere with coitus, decreasing the desire to engage in sexual activity.)

In understanding and guiding a woman through her acceptance of pregnancy, a maternity nurse should be aware that: A. Nonacceptance of the pregnancy very often equates to rejection of the child. B. Mood swings most likely are the result of worries about finances and a changed lifestyle as well as profound hormonal changes. C. Ambivalent feelings during pregnancy usually are seen only in emotionally immature or very young mothers. D. Conflicts such as not wanting to be pregnant or childrearing and career-related decisions need not be addressed during pregnancy because they will resolve themselves naturally after birth.

B (Mood swings are natural and are likely to affect every woman to some degree. A woman may dislike being pregnant, refuse to accept it, and still love and accept the child. Ambivalent feelings about pregnancy are normal for mature or immature women, younger or older women. Conflicts such as not wanting to be pregnant or childrearing and career-related decisions need to be resolved. The baby ends the pregnancy but not all the issues.)

A pregnant woman at 18 weeks of gestation calls the clinic to report that she has been experiencing occasional backaches of mild-to-moderate intensity. The nurse would recommend that she: A. Do Kegel exercises. B. Do pelvic rock exercises. C. Use a softer mattress. D. Stay in bed for 24 hours.

B (Pelvic rock exercises may help stretch and strengthen the abdominal and lower back muscles and relieve low back pain. Kegel exercises increase the tone of the pelvic area, not the back. A softer mattress may not provide the support needed to maintain proper alignment of the spine and may contribute to back pain. Stretching and other exercises to relieve back pain should be performed several times a day.)

The nurse caring for a newly pregnant woman would advise her that ideally prenatal care should begin: A. Before the first missed menstrual period. B. After the first missed menstrual period. C. After the second missed menstrual period. D. After the third missed menstrual period.

B (Prenatal care ideally should begin soon after the first missed menstrual period. Regular prenatal visits offer opportunities to ensure the health of the expectant mother and her infant.)

The nurse should have knowledge of the purpose of the pinch test. It is used to: A. Check the sensitivity of the nipples. B. Determine whether the nipple is everted or inverted. C. Calculate the adipose buildup in the abdomen. D. See whether the fetus has become inactive.

B (The pinch test is used to determine whether the nipple is everted or inverted. Nipples must be everted to allow breastfeeding.)

If exhibited by an expectant father, what would be a warning sign of ineffective adaptation to his partner's first pregnancy? A. Views pregnancy with pride as a confirmation of his virility B. Consistently changes the subject when the topic of the fetus/newborn is raised C. Expresses concern that he might faint at the birth of his baby D. Experiences nausea and fatigue, along with his partner, during the first trimester

B (This is an expected view for an expectant father. Persistent refusal to talk about the fetus-newborn may be a sign of a problem and should be assessed further. This is an expected feeling for an expectant father. This is an expected finding with expectant fathers.)

Which statement about multifetal pregnancy is inaccurate? A. The expectant mother often develops anemia because the fetuses have a greater demand for iron. B. Twin pregnancies come to term with the same frequency as single pregnancies. C. The mother should be counseled to increase her nutritional intake and gain more weight. D. Backache and varicose veins often are more pronounced.

B (Twin pregnancies often end in prematurity. Serious efforts should be made to bring the pregnancy to term. A woman with a multifetal pregnancy often develops anemia, suffers more or worse backache, and needs to gain more weight. Counseling is needed to help her adjust to these conditions.)

To prevent gastrointestinal upset, clients should be instructed to take iron supplements: A) On a full stomach B) At bedtime C) After eating a meal D) With milk

B) At bedtime

Three servings of milk, yogurt, or cheese plus two servings of meat, poultry, or fish adequately supply the recommended amount of protein for a pregnant woman. Many patients are concerned about the increased levels of mercury in fish and may be afraid to include this source of nutrients in their diet. Sound advice by the nurse to assist the client in determining which fish is safe to consume would include: A) Canned white tuna is a preferred choice. B) Avoid shark, swordfish, and mackerel. C) Fish caught in local waterways are the safest. D) Salmon and shrimp contain high levels of mercury.

B) Avoid shark, swordfish, and mackerel.

With regard to nutritional needs during lactation, a maternity nurse should be aware that: A) The mothers intake of vitamin C, zinc, and protein now can be lower than during pregnancy. B) Caffeine consumed by the mother accumulates in the infant, who may be unusually active and wakeful. C) Critical iron and folic acid levels must be maintained. D) Lactating women can go back to their prepregnant calorie intake.

B) Caffeine consumed by the mother accumulates in the infant, who may be unusually active and wakeful.

In teaching the pregnant adolescent about nutrition, the nurse should: A) Emphasize the need to eliminate common teen snack foods because they are too high in fat and sodium. B) Determine the weight gain needed to meet adolescent growth and add 35 lb. C) Suggest that she not eat at fast-food restaurants to avoid foods of poor nutritional value. D) Realize that most adolescents are unwilling to make dietary changes during pregnancy.

B) Determine the weight gain needed to meet adolescent growth and add 35 lb.

A pregnant woman experiencing nausea and vomiting should: A) Drink a glass of water with a fat-free carbohydrate before getting out of bed in the morning. B) Eat small, frequent meals (every 2 to 3 hours). C) Increase her intake of high-fat foods to keep the stomach full and coated. D) Limit fluid intake throughout the day.

B) Eat small, frequent meals (every 2 to 3 hours).

The most important reason for evaluating the pattern of weight gain in pregnancy is to: A) Prevent excessive adipose tissue deposits B) Identify potential nutritional problems or complications of pregnancy C) Assess the need to limit caloric intake in obese women D) Determine cultural influences on the woman's diet

B) Identify potential nutritional problems or complications of pregnancy

Which statement made by a lactating woman would lead the nurse to believe that the woman might have lactose intolerance? A) I always have heartburn after I drink milk. B) If I drink more than a cup of milk, I usually have abdominal cramps and bloating. C) Drinking milk usually makes me break out in hives. D) Sometimes I notice that I have bad breath after I drink a cup of milk.

B) If I drink more than a cup of milk, I usually have abdominal cramps and bloating.

Women with an inadequate weight gain during pregnancy are at higher risk of giving birth to an infant with: A) Spina bifida B) Intrauterine growth restriction C) Diabetes mellitus D) Down syndrome

B) Intrauterine growth restriction

Maternal nutritional status is an especially significant factor of the many factors that influence the outcome of pregnancy because: A) It is very difficult to adjust because of peoples' ingrained eating habits B) It is an important preventative measure for a variety of problems C) Women love obsessing about their weight and diets D) A woman's preconception weight becomes irrelevant

B) It is an important preventative measure for a variety of problems

A couple has been counseled for genetic anomalies. They ask you, "What is karyotyping?" Your best response is: a. "Karyotyping will reveal if the baby's lungs are mature." b. "Karyotyping will reveal if your baby will develop normally." c. "Karyotyping will provide information about the gender of the baby and the number and structure of the chromosomes." d. "Karyotyping will detect any physical deformities the baby has."

C

A father and mother are carriers of phenylketonuria (PKU). Their 2-year-old daughter has PKU. The couple tells the nurse that they are planning to have a second baby. Because their daughter has PKU, they are sure that their next baby won't be affected. What response by the nurse is most accurate? a. "Good planning; you need to take advantage of the odds in your favor." b. "I think you'd better check with your doctor first." c. "You are both carriers, so each baby has a 25% chance of being affected." d. "The ultrasound indicates a boy, and boys are not affected by PKU."

C

A pregnant woman at 25 weeks' gestation tells the nurse that she dropped a pan last week and her baby jumped at the noise. Which response by the nurse is most accurate? a. "That must have been a coincidence; babies can't respond like that." b. "The fetus is demonstrating the aural reflex." c. "Babies respond to sound starting at about 24 weeks of gestation." d. "Let me know if it happens again; we need to report that to your midwife."

C

A woman's cousin gave birth to an infant with a congenital heart anomaly. The woman asks the nurse when such anomalies occur during development. Which response by the nurse is most accurate? a. "We don't really know when such defects occur." b. "It depends on what caused the defect." c. "They occur between the third and fifth weeks of development." d. "They usually occur in the first 2 weeks of development."

C

In practical terms regarding genetic health care, nurses should be aware that: a. Genetic disorders affect people of all socioeconomic backgrounds, races, and ethnic groups equally. b. Genetic health care is more concerned with populations than individuals. c. The most important of all nursing functions is providing emotional support to the family during counseling. d. Taking genetic histories is the province of large universities and medical centers.

C

Sally comes in for her first prenatal examination. This is her first child. She asks you (the nurse), "How does my baby get air inside my uterus?" The correct response is: a. "The baby's lungs work in utero to exchange oxygen and carbon dioxide." b. "The baby absorbs oxygen from your blood system." c. "The placenta provides oxygen to the baby and excretes carbon dioxide into your bloodstream." d. "The placenta delivers oxygen-rich blood through the umbilical artery to the baby's abdomen."

C

The nurse is providing genetic counseling for an expectant couple who already have a child with trisomy 18. The nurse should: a. Tell the couple they need to have an abortion within 2 to 3 weeks. b. Explain that the fetus has a 50% chance of having the disorder. c. Discuss options with the couple, including amniocentesis to determine whether the fetus is affected. d. Refer the couple to a psychologist for emotional support.

C

With regard to the development of the respiratory system, maternity nurses should be understand that: a. The respiratory system does not begin developing until after the embryonic stage. b. The infant's lungs are considered mature when the lecithin/sphingomyelin (L/S) ratio is 1:1, at about 32 weeks. c. Maternal hypertension can reduce maternal-placental blood flow, accelerating lung maturity. d. Fetal respiratory movements are not visible on ultrasound scans until at least 16 weeks.

C

An expectant father confides in the nurse that his pregnant wife, 10 weeks of gestation, is driving him crazy. "One minute she seems happy, and the next minute she is crying over nothing at all. Is there something wrong with her?" The nurse's BEST response would be: A. "This is normal behavior and should begin to subside by the second trimester." B. "She may be having difficulty adjusting to pregnancy; I will refer her to a counselor that I know." C. "This is called emotional liability and is related to hormone changes and anxiety during pregnancy. The mood swings will eventually subside as she adjusts to being pregnant." D. "You seem impatient with her. Perhaps this is precipitating her behavior."

C (Although this statement is appropriate, it does not answer the father's question. Mood swings are a normal finding in the first trimester; the woman does not need counseling. This is the most appropriate response since it gives an explanation and a time frame for when the mood swings may stop. This statement is judgmental and not appropriate.)

With regard to the initial visit with a client who is beginning prenatal care, nurses should be aware that: A. The first interview is a relaxed, get-acquainted affair in which nurses gather some general impressions. B. If nurses observe handicapping conditions, they should be sensitive and not inquire about them because the client will do that in her own time. C. Nurses should be alert to the appearance of potential parenting problems, such as depression or lack of family support. D. Because of legal complications, nurses should not ask about illegal drug use; that is left to physicians

C (Besides these potential problems, nurses need to be alert to the woman's attitude toward health care. The initial interview needs to be planned, purposeful, and focused on specific content. A lot of ground must be covered. Nurses must be sensitive to special problems, but they do need to inquire because discovering individual needs is important. People with chronic or handicapping conditions forget to mention them because they have adapted to them. Getting information on drug use is important and can be done confidentially. Actual testing for drug use requires the client's consent.)

During the first trimester the pregnant woman would be most motivated to learn about: A. Fetal development. B. Impact of a new baby on family members. C. Measures to reduce nausea and fatigue so she can feel better. D. Location of childbirth preparation and breastfeeding classes

C (Fetal development concerns are more apparent in the second trimester when the woman is feeling fetal movement. Impact of a new baby on the family would be appropriate topics for the second trimester when the fetus becomes "real" as its movements are felt and its heartbeat heard. During this trimester a woman works on the task of, "I am going to have a baby." During the first trimester a woman is egocentric and concerned about how she feels. She is working on the task of accepting her pregnancy. Motivation to learn about childbirth techniques and breastfeeding is greatest for most women during the third trimester as the reality of impending birth and becoming a parent is accepted. A goal is to achieve a safe passage for herself and her baby.)

In response to requests by the U.S. Public Health Service for new models of prenatal care, an innovative new approach to prenatal care known as centering pregnancy was developed. Which statement would accurately apply to the centering model of care? A. Group sessions begin with the first prenatal visit. B. At each visit, blood pressure, weight, and urine dipsticks are obtained by the nurse. C. Eight to 12 women are placed in gestational-age cohort groups. D. Outcomes are similar to those of traditional prenatal care.

C (Gestational age cohorts comprise the groups, with approximately 8 to 12 women in each group. This group remains intact throughout the pregnancy. Individual follow-up visits are scheduled as needed. Group sessions begin at 12 to 16 weeks of gestation and end with an early postpartum visit. Before group sessions the client has an individual assessment, physical examination, and history. At the beginning of each group meeting, clients measure their own blood pressure, weight, and urine dips and enter these in their record. Fetal heart rate assessment and fundal height are obtained by the nurse. Results evaluating this approach have been very promising. In a study of adolescent clients, there was a decrease in low-birth-weight infants and an increase in breastfeeding rates.)

With regard to a woman's reordering of personal relationships during pregnancy, the maternity nurse should understand that: A. Because of the special motherhood bond, a woman's relationship with her mother is even more important than with the father of the child. B. Nurses need not get involved in any sexual issues the couple has during pregnancy, particularly if they have trouble communicating them to each other. C. Women usually express two major relationship needs during pregnancy: feeling loved and valued and having the child accepted by the father. D. The woman's sexual desire is likely to be highest in the first trimester because of the excitement and because intercourse is physically easier.

C (Love and support help a woman feel better about her pregnancy. The most important person to the pregnant woman is usually the father. Nurses can facilitate communication between partners about sexual matters if, as is common, they are nervous about expressing their worries and feelings. The second trimester is the time when a woman's sense of well-being, along with certain physical changes, increases her desire for sex. Desire is decreased in the first and third trimesters.)

A pregnant woman demonstrates understanding of the nurse's instructions regarding relief of leg cramps if she: A. Wiggles and points her toes during the cramp. B. Applies cold compresses to the affected leg. C. Extends her leg and dorsiflexes her foot during the cramp. D. Avoids weight bearing on the affected leg during the cramp.

C (Pointing toes can aggravate rather than relieve the cramp. Application of heat is recommended. Extending the leg and dorsiflexing the foot is the appropriate relief for a leg cramp. Bearing weight on the affected leg can help to relieve the leg cramp, so it should not be avoided.)

A pregnant woman at 10 weeks of gestation jogs three or four times per week. She is concerned about the effect of exercise on the fetus. The nurse should inform her: A. "You don't need to modify your exercising any time during your pregnancy." B. "Stop exercising, because it will harm the fetus." C. "You may find that you need to modify your exercise to walking later in your pregnancy, around the seventh month." D. "Jogging is too hard on your joints; switch to walking now."

C (The nurse should inform the woman that she may need to reduce her exercise level as the pregnancy progresses. Physical activity promotes a feeling of well-being in pregnant women. It improves circulation, promotes relaxation and rest, and counteracts boredom. Typically, running should be replaced with walking around the seventh month of pregnancy. Simple measures should be initiated to prevent injuries, such as warm-up and stretching exercises to prepare the joints for more strenuous exercise.)

The nurse should be aware that the partner's main role in pregnancy is to: A. Provide financial support. B. Protect the pregnant woman from "old wives' tales." C. Support and nurture the pregnant woman. D. Make sure the pregnant woman keeps prenatal appointments.

C (The partner's main role in pregnancy is to nurture the pregnant woman and respond to her feelings of vulnerability. In older societies, the man enacted the ritual couvade. Changing cultural and professional attitudes have encouraged fathers' participation in the birth experience over the past 30 years.)

To determine the cultural influence on a patients diet, the nurse should first: A) Evaluate the patients weight gain during pregnancy B) Assess the socioeconomic status of the patient C) Discuss the four food groups with the patient D) Identify the food preferences and methods of food preparation common to that culture

D) Identify the food preferences and methods of food preparation common to that culture

To provide the patient with accurate information about dental care during pregnancy, maternity nurses should be aware that: A. Dental care can be dropped from the priority list because the woman has enough to worry about and is getting a lot of calcium anyway. B. Dental surgery, in particular, is contraindicated because of the psychologic stress it engenders. C. If dental treatment is necessary, the woman will be most comfortable with it in the second trimester. D. Dental care interferes with the expectant mother's need to practice conscious relaxation.

C (The second trimester is best for dental treatment because that is when the woman will be able to sit most comfortably in the dental chair. Dental care such as brushing with fluoride toothpaste is especially important during pregnancy because nausea during pregnancy may lead to poor oral hygiene. Emergency dental surgery is permissible, but the mother must clearly understand the risks and benefits. Conscious relaxation is useful, and it may even help the woman get through any dental appointments; it is not a reason to avoid them.)

With regard to follow-up visits for women receiving prenatal care, nurses should be aware that: A. The interview portions become more intensive as the visits become more frequent over the course of the pregnancy. B. Monthly visits are scheduled for the first trimester, every 2 weeks for the second trimester, and weekly for the third trimester. C. During the abdominal examination, the nurse should be alert for supine hypotension. D. For pregnant women, a systolic blood pressure (BP) of 130 and a diastolic BP of 80 is sufficient to be considered hypertensive.

C (The woman lies on her back during the abdominal examination, possibly compressing the vena cava and aorta, which can cause a decrease in blood pressure and a feeling of faintness. The interview portion of follow-up examinations is less extensive than in the initial prenatal visits, during which so much new information must be gathered. Monthly visits are routinely scheduled for the first and second trimesters; visits increase to every 2 weeks at week 28 and to once a week at week 36. For pregnant women hypertension is defined as a systolic BP of 140 or greater and a diastolic BP of 90 or greater.)

A pregnant woman at 10 weeks of gestation jogs three or four times per week. She is concerned about the effect of exercise on the fetus. The nurse should inform her: A. "You don't need to modify your exercising any time during your pregnancy." B. "Stop exercising because it will harm the fetus." C. "You may find that you need to modify your exercise to walking later in your pregnancy, around the seventh month." D. "Jogging is too hard on your joints; switch to walking now."

C (Typically running should be replaced with walking around the seventh month of pregnancy. The nurse should inform the woman that she may need to reduce her exercise level as the pregnancy progresses. Physical activity promotes a feeling of well-being in pregnant women. It improves circulation, promotes relaxation and rest, and counteracts boredom. Simple measures should be initiated to prevent injuries, such as warm-up and stretching exercises to prepare the joints for more strenuous exercise.)

A 27-year-old pregnant woman had a preconceptual body mass index (BMI) of 18.0. The nurse knows that this woman's total recommended weight gain during pregnancy should be at least: A) 20 kg (44 lb) B) 16 kg (35 lb) C) 12.5 kg (27.5 lb) D) 10 kg (22 lb)

C) 12.5 kg (27.5 lb)

If a patients normal prepregnancy diet contains 45 g of protein daily, how many more grams of protein should she consume per day during pregnancy? A) 5 B) 10 C) 25 D) 30

C) 25

Which meal would provide the most absorbable iron? A) Toasted cheese sandwich, celery sticks, tomato slices, and a grape drink B) Oatmeal, whole wheat toast, jelly, and low-fat milk C) Black bean soup, wheat crackers, orange sections, and prunes D) Red beans and rice, cornbread, mixed greens, and decaffeinated tea

C) Black bean soup, wheat crackers, orange sections, and prunes

When providing care to the prenatal patient, the nurse understands that pica is defined as: A) Intolerance of milk products B) Iron deficiency anemia C) Ingestion of nonfood substances D) Episodes of anorexia and vomiting

C) Ingestion of nonfood substances

Which minerals and vitamins usually are recommended to supplement a pregnant womans diet? A) Fat-soluble vitamins A and D B) Water-soluble vitamins C and B6 C) Iron and Folate D) Calcium and zinc

C) Iron and Folate

Pregnant adolescents are at high risk for _____ because of lower body mass indices (BMIs) and fad dieting. A) Obesity B) Diabetes C) Low-birth-weight babies D) High-birth-weight babies

C) Low-birth-weight babies

While taking a diet history, the nurse might be told that the expectant mother has cravings for ice chips, cornstarch, and baking soda. This represents a nutritional problem known as: A) Preeclampsia B) Pyrosis C) Pica D) Purging

C) Pica

The labor and delivery nurse is preparing a bariatric patient for an elective cesarean birth. Which piece of specialized equipment is unnecessary when providing care for this pregnant woman. A) Extra long surgical instruments B) Wide surgical table C) Temporal thermometer D) Increased diameter blood pressure cuff

C) Temporal thermometer

A pregnant woman's diet consists almost entirely of whole grain breads and cereals, fruits, and vegetables. The nurse would be most concerned about this woman's intake of: A) Calcium B) Protein C) Vitamin B12 D) Folic acid

C) Vitamin B12

A woman who is 8 months pregnant asks the nurse, "Does my baby have any antibodies to fight infection?" The most appropriate response by the nurse is: a. "Your baby has all the immune globulins necessary: IgG, IgM, and IgA." b. "Your baby won't receive any antibodies until he is born and you breastfeed him." c. "Your baby does not have any antibodies to fight infection." d. "Your baby has IgG and IgM."

D

Some of the embryo's intestines remain within the umbilical cord during the embryonic period because the: a. Umbilical cord is much larger at this time than it will be at the end of pregnancy. b. Intestines begin their development within the umbilical cord. c. Nutrient content of the blood is higher in this location. d. Abdomen is too small to contain all the organs while they are developing.

D

The _____ is/are responsible for oxygen and carbon dioxide transport to and from the maternal bloodstream. a. Decidua basalis b. Germ layer c. Blastocyst d. Chorionic villi

D

The most basic information a maternity nurse should have concerning conception is that: a. Ova are considered fertile 48 to 72 hours after ovulation. b. Sperm remain viable in the woman's reproductive system for an average of 12 to 24 hours. c. Conception is achieved when a sperm successfully penetrates the membrane surrounding the ovum. d. Implantation in the endometrium occurs 6 to 10 days after conception.

D

With regard to the estimation and interpretation of the recurrence of risks for genetic disorders, nurses should be aware that: a. With a dominant disorder, the likelihood of the second child also having the condition is 100%. b. An autosomal recessive disease carries a one in eight risk of the second child also having the disorder. c. Disorders involving maternal ingestion of drugs carry a one in four chance of being repeated in the second child. d. The risk factor remains the same no matter how many affected children are already in the family.

D

The phenomenon of someone other than the mother-to-be experiencing pregnancy-like symptoms such as nausea and weight gain applies to the: A. Mother of the pregnant woman. B. Sister of the pregnant woman. C. Couple's teenage daughter. D. Expectant father.

D (An expectant father's experiencing pregnancy-like symptoms is called the couvade syndrome.)

Nutrition is one of the most significant factors influencing the outcome of a pregnancy. It is an alterable and important preventive measure for various potential problems, such as low birth weight and prematurity. While completing the physical assessment of the pregnant client, the nurse can evaluate the clients nutritional status by observing a number of physical signs. Which sign would indicate that the client has unmet nutritional needs? A) Normal heart rate, rhythm, and blood pressure B) Bright, clear, shiny eyes C) Alert, responsive, and good endurance D) Edema, tender calves, and tingling

D) Edema, tender calves, and tingling

A pregnant woman's diet may not meet her need for folates. A good source of this nutrient is: A) Chicken B) Cheese C) Potatoes D) Green leafy vegetables

D) Green leafy vegetables

While you are assessing the vital signs of a pregnant woman in her third trimester, the patient complains of feeling faint, dizzy, and agitated. Which nursing intervention is appropriate? A. Have the patient stand up and retake her blood pressure. B. Have the patient sit down and hold her arm in a dependent position. C. Have the patient lie supine for 5 minutes and recheck her blood pressure on both arms. D. Have the patient turn to her left side and recheck her blood pressure in 5 minutes.

D (Blood pressure is affected by maternal position during pregnancy. The supine position may cause occlusion of the vena cava and descending aorta. Turning the pregnant woman to a lateral recumbent position alleviates pressure on the blood vessels and quickly corrects supine hypotension. Pressures are significantly higher when the patient is standing. This option causes an increase in systolic and diastolic pressures. The arm should be supported at the same level of the heart. The supine position may cause occlusion of the vena cava and descending aorta, creating hypotension.)

A maternal serum alpha-fetoprotein (MSAFP) test is performed at 16 to 18 weeks of gestation. An elevated level has been associated with: A. Down syndrome. B. Sickle cell anemia. C. Cardiac defects. D. Open neural tube defects such as spina bifida.

D (Low levels of MSAFP are associated with Down syndrome. Sickle cell anemia is not detected by the MSAFP. Cardiac defects would not be detected with the MSAFP. A triple marker test determines the levels of MSAFP along with serum levels of estriol and human chorionic gonadotropin; an elevated level is associated with open neural tube defects.)

The nurse advises the woman who wants to have a nurse-midwife provide obstetric care that: A. She will have to give birth at home. B. She must see an obstetrician as well as the midwife during pregnancy. C. She will not be able to have epidural analgesia for labor pain. D. She must be having a low-risk pregnancy.

D (Most nurse-midwife births are managed in hospitals or birth centers; a few may be managed in the home. Nurse-midwives may practice with physicians or independently with an arrangement for physician backup. They must refer clients to physicians for complications. Care in a midwifery model is noninterventional, and the woman and family usually are encouraged to be active participants in the care. This does not imply that medications for pain control are prohibited. Midwives usually see low-risk obstetric clients. Care is often noninterventional with active involvement from the woman and her family. Nurse-midwives must refer clients to physicians for complications.)

When discussing work and travel during pregnancy with a pregnant patient, nurses should instruct them that: A. Women should sit for as long as possible and cross their legs at the knees from time to time for exercise. B. Women should avoid seat belts and shoulder restraints in the car because they press on the fetus. C. Metal detectors at airport security checkpoints can harm the fetus if the woman passes through them a number of times. D. While working or traveling in a car or on a plane, women should arrange to walk around at least every hour or so

D (Periodic walking helps prevent thrombophlebitis. Pregnant women should avoid sitting or standing for long periods and crossing the legs at the knees. Pregnant women must wear lap belts and shoulder restraints. The most common injury to the fetus comes from injury to the mother. Metal detectors at airport security checkpoints do not harm fetuses.)

A woman who is 14 weeks pregnant tells the nurse that she always had a glass of wine with dinner before she became pregnant. She has abstained during her first trimester and would like to know if it is safe for her to have a drink with dinner now. The nurse tells her: A. "Because you're in your second trimester, there's no problem with having one drink with dinner." B. "One drink every night is too much. One drink three times a week should be fine." C. "Because you're in your second trimester, you can drink as much as you like." D. "Because no one knows how much or how little alcohol it takes to cause fetal problems, the best course is to abstain throughout your pregnancy."

D (Regardless of which trimester the woman has reached, no amount of alcohol during pregnancy has been deemed safe for the fetus. Neither one drink per night nor three drinks per week is a safe recommendation. Although the first trimester is a crucial period of fetal development, pregnant women of all gestations are counseled to eliminate all alcohol from their diet. A safe level of alcohol consumption during pregnancy has not yet been established. Although the consumption of occasional alcoholic beverages may not be harmful to the mother or her developing fetus, complete abstinence is strongly advised.)

Which symptom is considered a first-trimester warning sign and should be reported immediately by the pregnant woman to her health care provider? A. Nausea with occasional vomiting B. Urinary frequency C. Fatigue D. Vaginal bleeding

D (Signs and symptoms that must be reported include severe vomiting, fever and chills, burning on urination, diarrhea, abdominal cramping, and vaginal bleeding. These symptoms may be signs of potential complications of the pregnancy. Nausea with occasional vomiting, fatigue, and urinary frequency are normal first-trimester complaints. Although they may be worrisome or annoying to the mother, they usually are not indications of pregnancy problems.)

An expectant couple asks the nurse about intercourse during pregnancy and if it is safe for the baby. The nurse should tell the couple that: A. Intercourse should be avoided if any spotting from the vagina occurs afterward. B. Intercourse is safe until the third trimester. C. Safer-sex practices should be used once the membranes rupture. D. Intercourse and orgasm are often contraindicated if a history or signs of preterm labor are present.

D (Some spotting can normally occur as a result of the increased fragility and vascularity of the cervix and vagina during pregnancy. Intercourse can continue as long as the pregnancy is progressing normally. Safer-sex practices are always recommended; rupture of the membranes may require abstaining from intercourse. Uterine contractions that accompany orgasm can stimulate labor and would be problematic if the woman were at risk for or had a history of preterm labor.)

In her work with pregnant women of various cultures, a nurse practitioner has observed various practices that seemed strange or unusual. She has learned that cultural rituals and practices during pregnancy seem to have one purpose in common. Which statement best describes that purpose? A. To promote family unity B. To ward off the "evil eye" C. To appease the gods of fertility D. To protect the mother and fetus during pregnancy

D (The purpose of all cultural practices is to protect the mother and fetus during pregnancy. Although many cultures consider pregnancy normal, certain practices are expected of women of all cultures to ensure a good outcome. Cultural prescriptions tell women what to do, and cultural proscriptions establish taboos. The purposes of these practices are to prevent maternal illness resulting from a pregnancy-induced imbalanced state and to protect the vulnerable fetus.)

A woman who is 14 weeks pregnant tells the nurse that she always had a glass of wine with dinner before she became pregnant. She has abstained during her first trimester and would like to know if it is safe for her to have a drink with dinner now. The nurse would tell her: A. "Since you're in your second trimester, there's no problem with having one drink with dinner." B. "One drink every night is too much. One drink three times a week should be fine." C. "Since you're in your second trimester, you can drink as much as you like." D. "Because no one knows how much or how little alcohol it takes to cause fetal problems, the best course is to abstain throughout your pregnancy."

D (The statement "Because no one knows how much or how little alcohol it takes to cause fetal problems, the best course is to abstain throughout your pregnancy" is accurate. A safe level of alcohol consumption during pregnancy has not yet been established. Although the consumption of occasional alcoholic beverages may not be harmful to the mother or her developing fetus, complete abstinence is strongly advised.)

A woman is 3 months pregnant. At her prenatal visit, she tells the nurse that she doesn't know what is happening; one minute she's happy that she is pregnant, and the next minute she cries for no reason. Which response by the nurse is most appropriate? A. "Don't worry about it; you'll feel better in a month or so." B. "Have you talked to your husband about how you feel?" C. "Perhaps you really don't want to be pregnant." D. "Hormonal changes during pregnancy commonly result in mood swings."

D (The statement "Hormonal changes during pregnancy commonly result in mood swings" is accurate and the most appropriate response by the nurse. The statement "Don't worry about it; you'll feel better in a month or so" dismisses the client's concerns and is not the most appropriate response. Although women should be encouraged to share their feelings, "Have you talked to your husband about how you feel" is not the most appropriate response and does not provide the client with a rationale for the psychosocial dynamics of her pregnancy. "Perhaps you really don't want to be pregnant" is completely inappropriate and deleterious to the psychologic well-being of the woman. Hormonal and metabolic adaptations often cause mood swings in pregnancy. The woman's responses are normal. She should be reassured about her feelings.)

What represents a typical progression through the phases of a woman's establishing a relationship with the fetus? A. Accepts the fetus as distinct from herself—accepts the biologic fact of pregnancy—has a feeling of caring and responsibility B. Fantasizes about the child's gender and personality—views the child as part of herself—becomes introspective C. Views the child as part of herself—has feelings of well-being—accepts the biologic fact of pregnancy D. "I am pregnant."—"I am going to have a baby."—"I am going to be a mother.

D (The woman first centers on herself as pregnant, then on the baby as an entity separate from herself, and then on her responsibilities as a mother. The expressions, "I am pregnant," "I am going to have a baby," and "I am going to be a mother" sum up the progression through the three phases.)

What laboratory results would be a cause for concern if exhibited by a woman at her first prenatal visit during the second month of her pregnancy? A. Hematocrit 38%, hemoglobin 13 g/dL B. White blood cell count 6000/mm3 C. Platelets 300,000/mm3 D. Rubella titer 1:6

D (This is a normal laboratory value in the pregnant woman. This is a normal laboratory value in the pregnant woman. This is a normal laboratory value in the pregnant woman. A rubella titer of less than 1:10 indicates a lack of immunity to rubella, a viral infection that has the potential to cause teratogenic effects on fetal development. Arrangements should be made to administer the rubella vaccine after birth during the postpartum period since administration of rubella, a live vaccine, would be contraindicated during pregnancy. Women receiving the vaccine during the postpartum period should be cautioned to avoid pregnancy for 3 months.)

A pregnant woman at 32 weeks of gestation complains of feeling dizzy and light-headed while her fundal height is being measured. Her skin is pale and moist. The nurse's initial response would be to: A. Assess the woman's blood pressure and pulse. B. Have the woman breathe into a paper bag. C. Raise the woman's legs. D. Turn the woman on her side.

D (Vital signs can be assessed next. Breathing into a paper bag is the solution for dizziness related to respiratory alkalosis associated with hyperventilation. Raising her legs will not solve the problem since pressure will still remain on the major abdominal blood vessels, thereby continuing to impede cardiac output. During a fundal height measurement the woman is placed in a supine position. This woman is experiencing supine hypotension as a result of uterine compression of the vena cava and abdominal aorta. Turning her on her side will remove the compression and restore cardiac output and blood pressure.)

For what reason would breastfeeding be contraindicated? A. Hepatitis B B. Everted nipples C. History of breast cancer 3 years ago D. Human immunodeficiency virus (HIV) positive

D (Women who are HIV positive are discouraged from breastfeeding. Although hepatitis B antigen has not been shown to be transmitted through breast milk, as an added precaution infants born to HBsAg-positive women should receive the hepatitis B vaccine and immune globulin immediately after birth. Everted nipples are functional for breastfeeding. Newly diagnosed breast cancer would be a contraindication to breastfeeding.)

A pregnant womans diet history indicates that she likes the following list of foods. The nurse would encourage this woman to consume more of which food to increase her calcium intake? A) Fresh apricots B) Canned clams C) Spaghetti with meat sauce D) Canned sardines

D) Canned sardines

The major source of nutrients in the diet of a pregnant woman should be composed of: A) Simple sugars B) Fats C) Fiber D) Complex carbohydrates

D) Complex carbohydrates

When counseling a client about getting enough iron in her diet, the maternity nurse should tell her that: A) Milk, coffee, and tea aid iron absorption if consumed at the same time as iron. B) Iron absorption is inhibited by a diet rich in vitamin C. C) Iron supplements are permissible for children in small doses. D) Constipation is common with iron supplements.

D) Constipation is common with iron supplements.

A woman in week 34 of pregnancy reports that she is very uncomfortable because of heartburn. The nurse would suggest that the woman: A) Substitute other calcium sources for milk in her diet. B) Lie down after each meal. C) Reduce the amount of fiber she consumes. D) Eat five small meals daily.

D) Eat five small meals daily.

A woman has come to the clinic for preconception counseling because she wants to start trying to get pregnant in 3 months. She can expect the following advice: A) Discontinue all contraception now B) Lose weight so that you can gain more during pregnancy C) You may take any medications you have been taking regularly D) Make sure that you include adequate folic acid in your diet

D) Make sure that you include adequate folic acid in your diet

A 22-year-old woman pregnant with a single fetus has a preconception body mass index (BMI) of 24. When she was seen in the clinic at 14 weeks of gestation, she had gained 1.8 kg (4 lb) since conception. How would the nurse interpret this? A) This weight gain indicates possible gestational hypertension. B) This weight gain indicates that the woman's infant is at risk for intrauterine growth restriction (IUGR). C) This weight gain cannot be evaluated until the woman has been observed for several more weeks. D) The woman's weight gain is appropriate for this stage of pregnancy.

D) The woman's weight gain is appropriate for this stage of pregnancy.

Which vitamins or minerals can lead to congenital malformations of the fetus if taken in excess by the mother? A) Zinc B) Vitamin D C) Folic Acid D) Vitamin A

D) Vitamin A

Which pregnant woman should restrict her weight gain during pregnancy? A) Woman pregnant with twins B) Woman in early adolescence C) Woman shorter than 62 inches or 157 cm D) Woman who was 20 pounds overweight before pregnancy

D) Woman who was 20 pounds overweight before pregnancy

2. A 39-year-old primigravida thinks that she is about 8 weeks pregnant, although she has had irregular menstrual periods all her life. She has a history of smoking approximately one pack of cigarettes a day, but she tells you that she is trying to cut down. Her laboratory data are within normal limits. What diagnostic technique could be used with this pregnant woman at this time? a. Ultrasound examination b. Maternal serum alpha-fetoprotein (MSAFP) screening c. Amniocentesis d. Nonstress test (NST)

a. Ultrasound examination

13. Nurses should be aware that the biophysical profile (BPP): a. Is an accurate indicator of impending fetal death. b. Is a compilation of health risk factors of the mother during the later stages of pregnancy. c. Consists of a Doppler blood flow analysis and an amniotic fluid index. d. Involves an invasive form of ultrasound examination.

a. Is an accurate indicator of impending fetal death.

22. Which analysis of maternal serum may predict chromosomal abnormalities in the fetus? a. Multiple-marker screening b. Lecithin/sphingomyelin (L/S) ratio c. Biophysical profile d. Type and crossmatch of maternal and fetal serum

a. Multiple-marker screening

21. A pregnant woman's biophysical profile score is 8. She asks the nurse to explain the results. The nurse's best response is: a. "The test results are within normal limits." b. "Immediate delivery by cesarean birth is being considered." c. "Further testing will be performed to determine the meaning of this score." d. "An obstetric specialist will evaluate the results of this profile and, within the next week, will inform you of your options regarding delivery."

a. "The test results are within normal limits."

8. A client asks her nurse, "My doctor told me that he is concerned with the grade of my placenta because I am overdue. What does that mean?" The best response by the nurse is: a. "Your placenta changes as your pregnancy progresses, and it is given a score that indicates the amount of calcium deposits it has. The more calcium deposits, the higher the grade, or number, that is assigned to the placenta. It also means that less blood and oxygen can be delivered to your baby." b. "Your placenta isn't working properly, and your baby is in danger." c. "This means that we will need to perform an amniocentesis to detect if you have any placental damage." d. "Don't worry about it. Everything is fine."

a. "Your placenta changes as your pregnancy progresses, and it is given a score that indicates the amount of calcium deposits it has. The more calcium deposits, the higher the grade, or number, that is assigned to the placenta. It also means that less blood and oxygen can be delivered to your baby."

3. The nurse sees a woman for the first time when she is 30 weeks pregnant. The woman has smoked throughout the pregnancy, and fundal height measurements now are suggestive of growth restriction in the fetus. In addition to ultrasound to measure fetal size, what other tool would be useful in confirming the diagnosis? a. Doppler blood flow analysis b. Contraction stress test (CST) c. Amniocentesis d. Daily fetal movement counts

a. Doppler blood flow analysis

16. Compared with contraction stress test (CST), nonstress test (NST) for antepartum fetal assessment: a. Has no known contraindications. b. Has fewer false-positive results. c. Is more sensitive in detecting fetal compromise. d. Is slightly more expensive.

a. Has no known contraindications.

4. A 41-week pregnant multigravida presents in the labor and delivery unit after a nonstress test indicated that her fetus could be experiencing some difficulties in utero. Which diagnostic tool would yield more detailed information about the fetus? a. Ultrasound for fetal anomalies b. Biophysical profile (BPP) c. Maternal serum alpha-fetoprotein (MSAFP) screening d. Percutaneous umbilical blood sampling (PUBS)

b. Biophysical profile (BPP)

18. A woman has been diagnosed with a high risk pregnancy. She and her husband come into the office in a very anxious state. She seems to be coping by withdrawing from the discussion, showing declining interest. The nurse can best help the couple by: a. Telling her that the physician will isolate the problem with more tests. b. Encouraging her and urging her to continue with childbirth classes. c. Becoming assertive and laying out the decisions the couple needs to make. d. Downplaying her risks by citing success rate studies.

b. Encouraging her and urging her to continue with childbirth classes.

24. Which nursing intervention is necessary before a second-trimester transabdominal ultrasound? a. Place the woman NPO for 12 hours. b. Instruct the woman to drink 1 to 2 quarts of water. c. Administer an enema. d. Perform an abdominal preparation.

b. Instruct the woman to drink 1 to 2 quarts of water.

12. In the first trimester, ultrasonography can be used to gain information on: a. Amniotic fluid volume. b. Location of Gestational sacs c. Placental location and maturity. d. Cervical length.

b. Location of Gestational sacs

20. Risk factors tend to be interrelated and cumulative in their effect. While planning the care for a laboring client with diabetes mellitus, the nurse is aware that she is at a greater risk for: a. Oligohydramnios. b. Polyhydramnios. c. Postterm pregnancy. d. Chromosomal abnormalities.

b. Polyhydramnios.

7. Maternal serum alpha-fetoprotein (MSAFP) screening indicates an elevated level. MSAFP screening is repeated and again is reported as higher than normal. What would be the next step in the assessment sequence to determine the well-being of the fetus? a. Percutaneous umbilical blood sampling (PUBS) b. Ultrasound for fetal anomalies c. Biophysical profile (BPP) for fetal well-being d. Amniocentesis for genetic anomalies

b. Ultrasound for fetal anomalies

23. While working with the pregnant woman in her first trimester, the nurse is aware that chorionic villus sampling (CVS) can be performed during pregnancy at: a. 4 weeks b. 8 weeks c. 10 weeks d. 14 weeks

c. 10 weeks

5. At 35 weeks of pregnancy a woman experiences preterm labor. Tocolytics are administered and she is placed on bed rest, but she continues to experience regular uterine contractions, and her cervix is beginning to dilate and efface. What would be an important test for fetal well-being at this time? a. Percutaneous umbilical blood sampling (PUBS) b. Ultrasound for fetal size c. Amniocentesis for fetal lung maturity d. Nonstress test (NST)

c. Amniocentesis for fetal lung maturity

19. In the past, factors to determine whether a woman was likely to develop a high risk pregnancy were evaluated primarily from a medical point of view. A broader, more comprehensive approach to high-risk pregnancy has been adopted today. There are now four categories based on threats to the health of the woman and the outcome of pregnancy. Which of the following is not one of these categories? a. Biophysical b. Psychosocial c. Geographic d. Environmental

c. Geographic

17. The nurse providing care for the antepartum woman should understand that contraction stress test (CST): a. Sometimes uses vibroacoustic stimulation. b. Is an invasive test; however, contractions are stimulated. c. Is considered negative if no late decelerations are observed with the contractions. d. Is more effective than nonstress test (NST) if the membranes have already been ruptured.

c. Is considered negative if no late decelerations are observed with the contractions.

10. When nurses help their expectant mothers assess the daily fetal movement counts, they should be aware that: a. Alcohol or cigarette smoke can irritate the fetus into greater activity. b. "Kick counts" should be taken every half hour and averaged every 6 hours, with every other 6-hour stretch off. c. The fetal alarm signal should go off when fetal movements stop entirely for 12 hours. d. Obese mothers familiar with their bodies can assess fetal movement as well as average-size women.

c. The fetal alarm signal should go off when fetal movements stop entirely for 12 hours.

14. With regard to amniocentesis, nurses should be aware that: a. Because of new imaging techniques, amniocentesis is now possible in the first trimester. b. Despite the use of ultrasound, complications still occur in the mother or infant in 5% to 10% of cases. c. The shake test, or bubble stability test, is a quick means of determining fetal maturity. d. The presence of meconium in the amniotic fluid is always cause for concern.

c. The shake test, or bubble stability test, is a quick means of determining fetal maturity.

1. A woman arrives at the clinic seeking confirmation that she is pregnant. The following information is obtained: She is 24 years old with a body mass index (BMI) of 17.5. She admits to having used cocaine "several times" during the past year and drinks alcohol occasionally. Her blood pressure (BP) is 108/70 mm Hg, her pulse rate is 72 beats/min, and her respiratory rate is 16 breaths/min. The family history is positive for diabetes mellitus and cancer. Her sister recently gave birth to an infant with a neural tube defect (NTD). Which characteristics place the woman in a high risk category? a. Blood pressure, age, BMI b. Drug/alcohol use, age, family history c. Family history, blood pressure, BMI d. Family history, BMI, drug/alcohol abuse

d. Family history, BMI, drug/alcohol abuse

15. Nurses should be aware of the strengths and limitations of various biochemical assessments during pregnancy, including that: a. Chorionic villus sampling (CVS) is becoming more popular because it provides early diagnosis. b. Maternal serum alpha-fetoprotein (MSAFP) screening is recommended only for women at risk for neural tube defects. c. Percutaneous umbilical blood sampling (PUBS) is one of the triple-marker tests for Down syndrome. d. MSAFP is a screening tool only; it identifies candidates for more definitive procedures.

d. MSAFP is a screening tool only; it identifies candidates for more definitive procedures.

25. The nurse recognizes that a nonstress test (NST) in which two or more fetal heart rate (FHR) accelerations of 15 beats/min or more occur with fetal movement in a 20-minute period is: a. Nonreactive b. Positive c. Negative d. Reactive

d. Reactive

11. In comparing the abdominal and transvaginal methods of ultrasound examination, nurses should explain to their clients that: a. Both require the woman to have a full bladder. b. The abdominal examination is more useful in the first trimester. c. Initially the transvaginal examination can be painful. d. The transvaginal examination allows pelvic anatomy to be evaluated in greater detail.

d. The transvaginal examination allows pelvic anatomy to be evaluated in greater detail.

6. A 40-year-old woman is 10 weeks pregnant. Which diagnostic tool would be appropriate to suggest to her at this time? a. Biophysical profile (BPP) b. Amniocentesis c. Maternal serum alpha-fetoprotein (MSAFP) screening d. Transvaginal ultrasound

d. Transvaginal ultrasound


संबंधित स्टडी सेट्स

Ch.3 Designing a Web Page with CSS

View Set

Chapter 50: Antineoplastic Drugs

View Set